Search results

  • The '''order of operations''' is a [[mathematical convention]] for [[arithmetic]] computation. The below li ...m PEMDAS. An AoPS mnemonic you can use to remember the order of operations is "Please Evaluate, My Dear AoPS Students".
    2 KB (271 words) - 13:19, 5 March 2022
  • An '''octahedron''' is a type of [[polyhedron]]. ...eometry]], an octahedron is any polyhedron with eight [[face]]s. The term is most frequently to refer to a polyhedron with eight [[triangular]] faces, w
    1 KB (155 words) - 12:19, 21 July 2023
  • ...f the [[circle]] that [[circumscribe]]s the triangle. Since every triangle is [[cyclic]], every triangle has a circumscribed circle, or a [[circumcircle] ...rea of the triangle. Then, the measure of the circumradius of the triangle is simply <math>R=\frac{abc}{4A}</math>. This can be rewritten as <math>A=\fra
    4 KB (729 words) - 16:52, 19 February 2024
  • If <math>p</math> is a positive integer and <math>x</math> and <math>y</math> are real numbers, <math>x^3-y^3=(x-y)(x^2+xy+y^2)</math>
    3 KB (450 words) - 06:25, 6 May 2024
  • ...style are items that are set in the preamble. Often, much of the preamble is placed in a separate file and included using the \usepackage statement. Th ...defaults. One option that might need modification is the font size, which is 10pt by default but can be increased to 11pt or 12pt. A reference on other
    30 KB (5,171 words) - 10:16, 4 April 2021
  • ...ds or a basic preamble that you can include in any of your LaTeX documents is simple. Just follow these steps: ...X 2.9\tex\latex'''. (If you have an older MiKTeX installation, this folder is probably '''C:\texmf\tex\latex'''.)</li>
    9 KB (1,544 words) - 06:05, 24 February 2021
  • | <math>a^{i+1}_3</math>||a^{i+1}_3||<math>x^{3^2}</math>||x^{3^2} |<math>\sqrt{3}</math>||\sqrt{3}
    12 KB (1,898 words) - 15:31, 22 February 2024
  • An '''imaginary number''' is a [[complex number]] whose [[real part]] is equal to 0. In the [[complex plane]], these numbers lie on the [[imaginary ...= \sqrt{-1}</math> is the [[imaginary unit]] and <math>\textrm{Im}</math> is the [[imaginary part]] [[function]].
    2 KB (283 words) - 00:18, 8 March 2012
  • '''Science Olympiad''' is a national team-based science competition primarily for middle high school ...school is allowed to have up to five ninth graders on its team. Division C is generally for high school students in grades 10-12, though ninth graders ar
    3 KB (443 words) - 11:27, 25 May 2023
  • ...e form <math>am + bn</math> for [[nonnegative]] integers <math>a, b</math> is <math>mn-m-n</math>. ...t in each pair of the form <math>(k, mn-m-n-k)</math>, exactly one element is expressible.
    17 KB (2,748 words) - 19:22, 24 February 2024
  • ...dodecagon''' is a 12-sided [[polygon]]. The sum of its internal [[angle]]s is <math>1800^{\circ}</math>. Each of its exterior angles has measure <math>3 D=dir(360/12*3);
    1 KB (219 words) - 13:08, 15 June 2018
  • The <math>n^{th}</math> triangular number is the sum of all natural numbers from one to n. That is, the <math>n^{th}</math> triangle number is
    2 KB (275 words) - 08:39, 7 July 2021
  • ...math>Q(x)</math> does there exist a polynomial <math>R(x)</math> of degree 3 such that <math>P(Q(x))=P(x) \cdot R(x)</math>? <math>P(Q(x))=(Q(x)-1)(Q(x)-2)(Q(x)-3)=P(x)\cdot R(x)=(x-1)(x-2)(x-3)\cdot R(x)</math>.
    5 KB (833 words) - 18:17, 8 April 2024
  • ...}\}</math>. What is the [[probability]] that <math>\mathrm{log}_a b</math> is an [[integer]]? ...<math>\sum_{x=1}^{12} \lfloor\frac {25}x-1\rfloor = 24 + 11 + 7 + 5 + 4 + 3 + 2 + 2 + 1 + 1 + 1 + 1 = 62</math>.
    1 KB (175 words) - 20:35, 7 November 2013
  • ...with a rectangular base that measures 40 cm by 20 cm and a height of 10 cm is placed in the aquarium. By how many centimeters does the water rise? ...th>4000 cm^3</math>; therefore, when the volume increases by <math>8000 cm^3</math>, the water level rises <math>2 cm \Rightarrow\fbox{D}</math>
    895 bytes (140 words) - 21:30, 3 July 2013
  • The larger of two consecutive odd integers is three times the smaller. What is their sum? *Thus, the answer is <math>1+(1+2)=4 \mathrm{(A)}</math>
    585 bytes (81 words) - 02:09, 16 February 2021
  • ...measures 40 degrees, and angle <math>ADC</math> measures 140 degrees. What is the degree measure of angle <math>BAD</math>? Since triangle <math>ABC</math> is isosceles we know that angle <math>\angle BAC = \angle BCA</math>.
    2 KB (265 words) - 00:20, 30 October 2022
  • ...</math> has real coefficients, and <math>f(2i) = f(2 + i) = 0.</math> What is <math>a + b + c + d?</math> <math>x^4 - 4x^3 + 9x^2 - 16x + 20 = 0</math></div>
    2 KB (319 words) - 00:37, 25 March 2024
  • ...s fast as he walks, and both choices require the same amount of time. What is the [[ratio]] of Yan's distance from his home to his distance from the stad ...-1 = \frac{4}{3}</math>, so the answer is <math>\boxed{\mathrm{(B)}\ \frac{3}{4}}</math>
    5 KB (804 words) - 14:55, 21 August 2022
  • Direct proportion is a proportion in which one number is a multiple of the other. Direct proportion between two numbers <math>x</mat where <math>k</math> is some [[real number]].
    2 KB (325 words) - 16:34, 1 June 2022
  • '''Parity''' refers to whether a number is [[even]] or [[odd]]. While this may seem highly basic, checking the parity of numbers is often an useful tactic for solving problems, especially with [[proof by con
    4 KB (694 words) - 22:00, 12 January 2024
  • ...ouse starts getting farther from the cheese rather than closer to it. What is <math>a+b</math>? ...th>. The perpendicular has slope <math>\frac{1}{5}</math>, so its equation is <math>y=10+\frac{1}{5}(x-12)=\frac{1}{5}x+\frac{38}{5}</math>. The <math>x<
    2 KB (387 words) - 18:20, 27 November 2023
  • What is <math>a+b+c+d+e</math>? ...th>e</math> are <math>9, 7, 5, 3,</math> and <math>1,</math> and their sum is <math>\fbox{25 (C)}</math>
    2 KB (278 words) - 02:10, 16 February 2021
  • ...sum of the digits of <math>n.</math> For how many values of <math>n</math> is <math>n + S(n) + S(S(n)) = 2007?</math> <math>\mathrm{(A)}\ 1 \qquad \mathrm{(B)}\ 2 \qquad \mathrm{(C)}\ 3 \qquad \mathrm{(D)}\ 4 \qquad \mathrm{(E)}\ 5</math>
    15 KB (2,558 words) - 19:33, 4 February 2024
  • ...y = 2\log_{a}x,</math> and <math>y = 3\log_{a}x,</math> respectively. What is <math>a?</math> ...A)}\ \sqrt [6]{3}\qquad \mathrm{(B)}\ \sqrt {3}\qquad \mathrm{(C)}\ \sqrt [3]{6}\qquad \mathrm{(D)}\ \sqrt {6}\qquad \mathrm{(E)}\ 6</math>
    3 KB (548 words) - 18:09, 21 July 2020
  • ...ath>. If <math>x = 8</math> when <math>y = 1/2</math> and <math>z = \sqrt {3}/2</math>, find <math>y</math> when <math>x = 1</math> and <math>z = 6</mat ...the problem. Using the second gives us <math>1*y=y=4</math>, so the answer is <math>y=4</math>.
    1 KB (170 words) - 14:24, 26 February 2013
  • ...<math>\sin{x}=\sin{(nx)}</math> on the interval <math>[0,\pi]</math>. What is <math>\sum_{n=2}^{2007} F(n)</math>? <math>F(2)=3</math>
    4 KB (588 words) - 14:40, 23 August 2023
  • ...ee-digit numbers are composed of three distinct digits such that one digit is the [[average]] of the other two? We can find the number of increasing [[arithmetic sequence]]s of length 3 possible from 0 to 9, and then find all the possible permutations of these
    2 KB (336 words) - 05:01, 4 November 2022
  • ...n b = \sqrt{\frac{5}{3}}</math> and <math>\cos a + \cos b = 1</math>. What is <math>\cos (a - b)</math>? <math>\mathrm{(A)}\ \sqrt{\frac{5}{3}} - 1\qquad \mathrm{(B)}\ \frac 13\qquad \mathrm{(C)}\ \frac 12\qquad \math
    1,022 bytes (153 words) - 14:56, 7 August 2017
  • ...),</math> <math>D = (680,380),</math> and <math>E = (689,389).</math> What is the sum of all possible x coordinates of <math>A</math>? ...= \frac 12bh</math>, we have that the height of <math>\triangle ABC</math> is <math>h = \frac{2k}{b} = \frac{2007 \cdot 2}{223} = 18</math>. Thus <math>A
    4 KB (565 words) - 17:01, 2 April 2023
  • ...out of any three consecutive integers. How many [[subset]]s of <math>\{1,2,3,\ldots,12\},</math> including the [[empty set]], are spacy? ...ath>\{ 1, 2, ... n \}</math>. We have <math>S_{0} = 1, S_{1} = 2, S_{2} = 3</math>.
    9 KB (1,461 words) - 23:07, 27 January 2024
  • ...ber ''prime-looking'' if it is [[composite]] but not divisible by <math>2, 3,</math> or <math>5.</math> The three smallest prime-looking numbers are <ma ...ath>1000 - (168-3) - 1 - |S_2 \cup S_3 \cup S_5|</math> (note that <math>2,3,5</math> are primes).
    2 KB (277 words) - 18:15, 25 November 2020
  • A faulty car odometer proceeds from digit 3 to digit 5, always skipping the digit 4, regardless of position. If the odo ...r both the other two [[intersection]]s. The intersection of all three sets is just <math>2</math>. So we get:
    4 KB (536 words) - 21:18, 22 May 2023
  • ...\ 0 \qquad \mathrm{(B)} \ 1 \qquad \mathrm{(C)} \ 2 \qquad \mathrm{(D)} \ 3 \qquad \mathrm{(E)} \ 4</math> ...onent of <math>a</math> becomes huge, and since <math>a \ge 2</math> there is no way we can satisfy the second condition. Hence we have two ordered tripl
    991 bytes (153 words) - 21:20, 3 July 2013
  • ...C 12A Problems|2005 AMC 12A #2]] and [[2005 AMC 10A Problems|2005 AMC 10A #3]]}} ...+ 7 = 3</math> and <math>bx - 10 = - 2</math> have the same solution. What is the value of <math>b</math>?
    829 bytes (116 words) - 19:06, 25 December 2022
  • ...of [[radians]] in a circle. For a convincing proof that <math>\tau</math> is a better circle constant than <math>\pi</math>, see [http://www.tauday.com == Why <math>\tau</math> Is Better Than <math>\pi</math>==
    2 KB (386 words) - 21:18, 22 November 2021
  • ...ly one-fourth of the total number of faces of the unit cubes are red. What is <math>n</math>? (\mathrm {A}) \ 3 \qquad (\mathrm {B}) \ 4 \qquad (\mathrm {C})\ 5 \qquad (\mathrm {D}) \ 6 \
    693 bytes (107 words) - 21:19, 3 July 2013
  • <math>\mathrm {A} =\dfrac{4\sqrt{3}}{4}=\sqrt{3}</math> <math>\mathrm {lh}=\sqrt{2^2-1^2}=\sqrt{3}</math>
    591 bytes (84 words) - 20:30, 30 December 2012
  • What is <math>A</math>? (\mathrm {A}) \ 1 \qquad (\mathrm {B}) \ 2 \qquad (\mathrm {C})\ 3 \qquad (\mathrm {D}) \ 4 \qquad (\mathrm {E})\ 5
    635 bytes (96 words) - 21:19, 3 July 2013
  • ...^2 + ax + 8x + 9 = 0</math> has only one solution for <math>x</math>. What is the sum of these values of <math>a</math>? ...o <math>\pm 12 = a + 8 \Longrightarrow a = 4, -20</math>. The sum of these is <math>-20 + 4 = -16 \Rightarrow \mathrm{(A)}</math>.
    2 KB (375 words) - 02:45, 14 January 2021
  • How many three-digit numbers satisfy the property that the middle digit is the [[mean|average]] of the first and the last digits? ...<math>C</math> and is unique for each <math>(A,C)</math>. Thus our answer is <math>9 \cdot 5 \cdot 1 = 45 \implies E</math>.
    2 KB (266 words) - 00:59, 19 October 2020
  • (\mathrm {A}) \ 0 \qquad (\mathrm {B}) \ 2 \qquad (\mathrm {C})\ 3 \qquad (\mathrm {D}) \ 8 \qquad (\mathrm {E})\ 9 The slope of the line is<math>
    1 KB (224 words) - 20:18, 15 July 2021
  • numbers 3, 5, 6, 7 and 9, although not necessarily in that order. The sums of the arithmetic sequence, although not necessarily in that order. What is the middle
    3 KB (430 words) - 18:52, 11 July 2020
  • ...om with each dot equally likely to be chosen. The die is then rolled. What is the [[probability]] that the top face has an odd number of dots? There are <math>1 + 2 + 3 + 4 + 5 + 6 = 21</math> dots total. [[Casework]]:
    2 KB (245 words) - 20:07, 4 March 2024
  • ...rline{AB}</math> and <math>\overline{DE}</math> is a second diameter. What is the [[ratio]] of the area of <math>\triangle DCE</math> to the area of <mat dotfactor=3;
    14 KB (1,970 words) - 17:02, 18 August 2023
  • ...r along the dashed lines shown in Figure 2. This creates nine pieces. What is the volume of the piece that contains vertex <math>W</math>? path d= (0,0)--(3,13)--(13,13)--(10,0);
    2 KB (215 words) - 13:56, 19 January 2021
  • The sum of <math>49</math> consecutive integers is <math>7^5</math>. What is their [[median]]? <math>\text {(A)}\ 7 \qquad \text {(B)}\ 7^2\qquad \text {(C)}\ 7^3\qquad \text {(D)}\ 7^4\qquad \text {(E)}\ 7^5</math>
    925 bytes (130 words) - 17:28, 9 July 2020
  • ...st integer <math>k </math> such that <math>P </math> is divisible by <math>3^k .</math> ...irst <math>100</math> positive odd integers can be written as <math>1\cdot 3\cdot 5\cdot 7\cdots 195\cdot 197\cdot 199=\frac{1\cdot 2\cdots200}{2\cdot4\
    4 KB (562 words) - 18:37, 30 October 2020
  • ...b, </math> and <math> c </math> are positive integers and <math> b</math> is not divisible by the square of any prime. Find <math> a+b+c. </math> dotfactor=3;
    7 KB (1,067 words) - 12:23, 8 April 2024
  • ...the smallest positive integer <math>n</math> for which <math> S_n </math> is an integer. ...th>. Since we're looking for the smallest such <math>n</math>, the answer is <math>\boxed{063}</math>.
    2 KB (263 words) - 23:32, 28 February 2021
  • ...paper triangles. Two large triangles are considered distinguishable if it is not possible to place one on the other, using translations, rotations, and/ ...choices for the three outer triangles, if two are one color and the third is a different color.
    4 KB (695 words) - 10:37, 4 November 2023
  • .../math> and <math> r</math> are [[relatively prime]], and <math> q </math> is not [[divisibility | divisible]] by the [[perfect square | square]] of any pair A=(0,0),B=(-3^.5,-3),C=(3^.5,-3),D=13*expi(-2*pi/3),E1=11*expi(-pi/3),F=E1+D;
    6 KB (1,033 words) - 02:36, 19 March 2022
  • .../math>. Given that the [[probability]] of obtaining face <math> F </math> is <math> m/n, </math> where <math> m </math> and <math> n </math> are [[relat ...er die. 7 can be obtained by rolling a 2 and 5, 5 and 2, 3 and 4, or 4 and 3. Each has a probability of <math>\frac{1}{6} \cdot \frac{1}{6} = \frac{1}{3
    5 KB (712 words) - 12:10, 5 November 2023
  • ...h>p, q,</math> and <math>r</math> are positive integers and <math>r</math> is not divisible by the square of any prime, find <math>p+q+r.</math> ...n do the same thing to get <math>O_3s_1 = 4</math> and <math>s_1s = 4\sqrt{3}</math>.
    3 KB (553 words) - 10:45, 26 August 2015
  • ...frac{(2n+1) + (2(n+j)-1)}{2}\right) = j(2n+j)</math>. Thus, <math>j</math> is a factor of <math>N</math>. ...must also be odd. For every odd value of <math>j</math>, <math>2n+j</math> is also odd, making this case valid for all odd <math>j</math>. Looking at the
    4 KB (675 words) - 10:40, 14 July 2022
  • ...m </math> and <math> n </math> are positive integers and <math> n </math> is not divisible by the square of any prime, find <math> m+n.</math> ...of length <math>x, y</math> and <math>z</math>, and suppose this triangle is acute (so all [[altitude]]s are in the interior of the triangle).
    4 KB (725 words) - 17:18, 27 June 2021
  • ...361, </math> find the [[remainder]] when <math>\sum^{28}_{k=1} a_k </math> is divided by 1000. Define the sum as <math>s</math>. Since <math>a_n\ = a_{n + 3} - a_{n + 2} - a_{n + 1} </math>, the sum will be:
    3 KB (417 words) - 10:07, 12 October 2023
  • ...<math> (a_1,a_2,a_3,\ldots,a_{12}) </math> be a permutation of <math> (1,2,3,\ldots,12) </math> for which An example of such a permutation is <math> (6,5,4,3,2,1,7,8,9,10,11,12). </math> Find the number of such permutations.
    2 KB (384 words) - 14:12, 20 April 2024
  • ...time both <math>a</math> and <math>b</math> can have a 0 in the tens digit is when they are divisible by 100, which falls into the above category, so we ...</math> numbers in every hundred numbers that have a tens digit of 0 (this is true from 100 to 900), totaling <math>9 \cdot 9 = 81</math> such numbers; c
    7 KB (1,114 words) - 03:41, 12 September 2021
  • ...team <math> A </math> finishes with more points than team <math> B </math> is <math> m/n, </math> where <math> m </math> and <math> n </math> are relativ ...th>A</math> and <math>B</math> end with the same score in these five games is <math>1-2p</math>.
    6 KB (983 words) - 13:42, 8 December 2021
  • ...and <math>\angle F</math> are congruent. The area of the hexagonal region is <math>2116(\sqrt{2}+1).</math> Find <math>AB</math>. ...th>\log_{10} 75</math>, and <math>\log_{10} n</math>, where <math>n</math> is a positive integer. Find the number of possible values for <math>n</math>.
    8 KB (1,350 words) - 12:00, 4 December 2022
  • The following is a list of [[PMWC]] problems from the year 2005 What is the greatest possible number one can get by discarding <math>100</math> dig
    9 KB (1,449 words) - 20:49, 2 October 2020
  • is the square of an integer. ...>b'=\min(a,b), a'=\max(a,b)</math>. Thus, <math>a'^2-kb'a'+b'^2-k=0</math> is a quadratic in <math>a'</math>. Should there be another root, <math>c'</mat
    4 KB (720 words) - 12:26, 7 April 2024
  • Calculate: <math>\frac{1*2*3+2*4*6+3*6*9+4*8*12+5*10*15}{1*3*5+2*6*10+3*9*15+4*12*20+5*15*25}</math> dot((z.x+j -1 + i/2 ,z.y + i*sqrt(3)/2));
    11 KB (1,738 words) - 19:25, 10 March 2015
  • ...> different factors (including the number <math>1</math> and itself). What is the smallest possible value of <math>y</math>? ...^{6972593}-1</math>. What is the remainder when <math>2^{6972593}-1</math> is divided by <math>5</math>?
    11 KB (1,713 words) - 22:47, 13 July 2023
  • for(int j = 0; j < 3; ++j) draw(origin--(-1.3*3,0)--(-1.3*3,3)--(0,3)--cycle);
    4 KB (641 words) - 21:24, 21 April 2014
  • 1+2 &=& 3 \\ ...number in the <math>80</math>th row (e.g. the last number of the third row is <math>15</math>).
    882 bytes (140 words) - 19:07, 10 March 2015
  • <math>DEB</math> is a chord of a circle such that <math>DE=3</math> and <math>EB=5 .</math> Let <math>O</math> be the center of the circ == Problem 3 ==
    3 KB (519 words) - 08:58, 13 September 2012
  • ...given figure. Find the perimeter, in cm, of <math>ABCD</math> if its area is <math>6750\text{ cm}^2</math>. pair A=(0,0),B=(0,-2.5),C=(3,-2.5),D=(3,0);
    944 bytes (154 words) - 12:44, 13 August 2014
  • ...math>1</math> and <math>6</math>, <math>2</math> and <math>5</math>, <math>3</math> and <math>4</math> appear on opposite faces. When <math>2</math> dic What is the sum of these <math>4</math> products ?
    1 KB (241 words) - 14:35, 20 April 2014
  • for(int i = 0; i < 3; ++i){ draw((4/3,i+1)--(4/3,i)--(8/3,i+1)--(8/3,i));
    929 bytes (135 words) - 14:37, 20 April 2014
  • ...t of the first prime). Note that <math>9</math> may only map one time, but is mapped to from both <math>1</math> and <math>7</math>, so <math>9</math> mu ...math>7</math> from being the second digit. These together imply that <math>3,7,9</math> cannot be the first digit, so the second digit must be <math>1</
    2 KB (234 words) - 15:30, 3 July 2012
  • && 1 \left(\dfrac{1}{1}+\dfrac{1}{2}+\dfrac{1}{3}+\dfrac{1}{4}+\dfrac{1}{5}+\dfrac{1}{6}+\dfrac{1}{7}+\dfrac{1}{8}+\dfrac{1} &+& 3\left(\dfrac{1}{2}+\dfrac{1}{3}+\dfrac{1}{4}+\dfrac{1}{5}+\dfrac{1}{6}+\dfrac{1}{7}+\dfrac{1}{8}+\dfrac{1}
    1 KB (139 words) - 19:07, 10 March 2015
  • ...nd <math>c</math> is a five-digit positive integer (natural number). What is the number <math>c</math>? :''The following solution is non-rigorous''.
    938 bytes (136 words) - 08:56, 6 August 2019
  • ...<math>9</math> and the last number is a multiple of <math>11</math>. What is the first of these four numbers? n \equiv -3 &\equiv& 8 \pmod{11}\\\end{eqnarray*}</cmath>
    1 KB (202 words) - 19:07, 10 March 2015
  • Let ''a'',''b'', and ''c'' be the lengths of a triangle whose area is ''S''. Prove that <math>a^2 + b^2 + c^2 \ge 4S\sqrt{3}</math>
    3 KB (425 words) - 21:18, 20 August 2020
  • ...ath> such that the points <math>L,M,N</math> do not form a triangle.) What is the locus of point <math>G</math> as <math>A', B', C'</math> range independ ...y point on plane <math>\epsilon</math>. Thus, the locus of <math>G</math> is the plane parallel to <math>\epsilon</math> and halfway between the centroi
    2 KB (301 words) - 23:29, 18 July 2016
  • ...pell SOS wins. If all boxes are filled without producing SOS then the game is a draw. Prove that the second player has a winning strategy. ...S in <math>l + 3</math> and if <math>l < m</math> he fills an S in <math>l-3</math>.
    2 KB (433 words) - 13:35, 4 July 2013
  • &=f(3,1)\\ &=1 & \text{Thus the answer is}\mathrm{(E)}
    701 bytes (92 words) - 16:51, 23 November 2016
  • ...to the circle <math>(B,B\Delta )</math>, the [[area]] of the shaded region is ...{(B)}\ 4\left(\sqrt2-\frac{\pi}{3}\right)\qquad\mathrm{(C)}\ \frac{8\sqrt2-3\pi}{16}\qquad\mathrm{(D)}\ \frac{\pi}{8}\qquad\mathrm{(E)}\ \text{None of t
    1 KB (214 words) - 23:44, 22 December 2016
  • ...shown. Each three point set has the same probability of being chosen. What is the probability that the points lie on the same straight line? There are <math>\binom{9}{3}</math> ways to choose three points out of the 9 there. There are 8 combina
    1,017 bytes (143 words) - 14:14, 21 April 2021
  • ...f the acute angle formed by the two lines? (Note: <math>\pi</math> radians is <math>180</math> degrees.) ...he area of the unshaded region be <math>U</math>, and the acute angle that is formed by the two lines be <math>\theta</math>. We can set up two equations
    3 KB (476 words) - 03:50, 23 January 2023
  • ...</math> intersects side <math>\overline{AD}</math> at <math>E</math>. What is the length of <math>\overline{CE}</math>? ...frac{1}{2}</math>. Solving <math>EF=\frac{1}{2}</math>. Adding, the answer is <math>\frac{5}{2}</math>.
    5 KB (738 words) - 13:11, 27 March 2023
  • ...<math>ABCD</math> so that <math>\triangle BEF</math> is equilateral. What is the ratio of the area of <math>\triangle DEF</math> to that of <math>\trian unitsize(3 cm);
    4 KB (710 words) - 02:47, 18 April 2024
  • <cmath> (u + u^2 + u^3 + \cdots + u^8) + 10u^9 = (v + v^2 + v^3 + \cdots + v^{10}) + 10v^{11} = 8. </cmath> ...e, with proof, which of the two numbers, <math>u</math> or <math>v</math>, is larger.
    2 KB (300 words) - 19:16, 18 July 2016
  • A sequence of functions <math>\, \{f_n(x) \} \,</math> is defined recursively as follows: (Recall that <math>\sqrt {\makebox[5mm]{}}</math> is understood to represent the positive square root.) For each positive intege
    3 KB (386 words) - 20:47, 3 July 2013
  • Isabella's house has 3 bedrooms. Each bedroom is 12 feet long, 10 feet wide, and 8 feet high. Isabella must paint the walls ==Problem 3==
    12 KB (1,814 words) - 12:58, 19 February 2020
  • ...the result by 9. Instead, she subtracted 9 and then divided the result by 3, giving an answer of 43. What would her answer have been had she worked the ...orrect result is <math>\frac{138-3}{9}=\frac{135}{9}=15</math>. Our answer is <math>\boxed{\textbf{(A) }15}</math>.
    1 KB (163 words) - 12:46, 8 November 2021
  • Find the degree measure of an angle whose complement is 25% of its supplement. ...om the supplementary angle, we have <math>90^{\circ}</math> as <math>\frac{3}{4}</math> of the supplementary angle.
    1,019 bytes (140 words) - 19:14, 20 August 2019
  • A sequence of [[function]]s <math>\, \{f_n(x) \} \,</math> is defined [[recursion|recursively]] as follows: (Recall that <math>\sqrt {\makebox[5mm]{}}</math> is understood to represent the positive [[square root]].) For each positive in
    2 KB (322 words) - 19:14, 18 July 2016
  • <math>1 - 2 + 3 -4 + \cdots - 98 + 99 = </math> Which of the following statements is false?
    13 KB (1,945 words) - 18:28, 19 June 2023
  • '''Cramer's Rule''' is a method of solving systems of equations using [[matrix|matrices]]. ...>. Here, <math>A</math> is the coefficient matrix, <math>\mathbf{b}</math> is a column vector.
    2 KB (352 words) - 18:22, 11 October 2023
  • <cmath>\geq ^{Cheff} \frac{1}{3}(a+b+c)\left( \frac{1}{\sqrt{S+6bc}}+\frac{1}{\sqrt{S+6ca}}+\frac{1}{\sqrt{ <cmath>\geq^{AH} \frac{1}{3}(a+b+c) \left( \frac{9}{\sqrt{S+6bc}+\sqrt{S+6ca}+\sqrt{S+6ab}} \right)</cm
    7 KB (1,174 words) - 12:14, 3 August 2023
  • ...[[ring theory]], omega is a constant the represents <math>e^{\frac{2i\pi}{3}}</math>. ...never one item in the ring is taken to any power, another item in the ring is the result.
    613 bytes (96 words) - 15:54, 5 June 2020
  • ...rty that <math>N</math> is divisible by 11, and <math>\dfrac{N}{11}</math> is equal to the sum of the squares of the digits of <math>N</math>. ...digits in the odd positions minus the sum of digits in the even positions is divisible by <math>11</math>. Thus we get: <math>b = a + c</math> or <math>
    4 KB (751 words) - 05:01, 17 August 2022
  • If <math>x \neq 0</math>, then the LHS is defined and rewrites as follows: ...}</math> is imaginary. So the original inequality holds iff <math>x</math> is in <math>[-1/2, 0) \cup (0, 45/8).</math>
    2 KB (239 words) - 05:51, 7 April 2024
  • ...>, where <math>X</math> is any point of <math>AC</math> and <math>Y</math> is any point of <math>B'D'</math>; ...t unique points <math>X</math> and <math>Y</math> such that <math>P</math> is the midpoint of <math>XY</math>.
    2 KB (402 words) - 23:28, 18 July 2016
  • ...ution with an inscribed sphere tangent to the base of the cone. A cylinder is circumscribed about this sphere so that one of its bases lies in the base o <cmath>V_1 = \frac{1}{3}\pi R^2 h</cmath>
    7 KB (1,214 words) - 18:49, 29 January 2018
  • <math>1 - 2 + 3 -4 + \cdots - 98 + 99 = </math> ...-1) + \cdots + 99</math>, and since there are 49 pairs of terms the answer is <math>-49 + 99 = 50 \Rightarrow \mathrm{\textbf{(E)}}</math>.
    1 KB (170 words) - 21:35, 9 February 2024
  • ''t'' is the radius of the first line, and s[] stores the radius of subsequent lines ...f the angle mark. However, anglemark() makes a bit more abstract sense and is more flexible.
    4 KB (646 words) - 21:18, 26 March 2024
  • An '''excircle''' is a [[circle]] [[Tangent line|tangent]] to the extensions of two sides of a [ pair X=(0,0), Y=(10,0), Z=(3,6);
    5 KB (843 words) - 03:02, 1 July 2020
  • <math>\triangle ABC</math> is a triangle. Take points <math>D, E, F</math> on the perpendicular bisectors But this is clearly true, since D lies on the perpendicular bisector of BC, BD = DC.
    2 KB (347 words) - 08:13, 26 August 2021
  • ...D_2</math> at two points, the closer of which to the vertex <math>A</math> is denoted by <math>Q</math>. Prove that <math>AQ = D_2P</math>. It is well known that the excircle opposite <math>A</math> is tangent to <math>\overline{BC}</math> at the point <math>D_2</math>. (Proof
    11 KB (2,091 words) - 08:35, 16 November 2017
  • In algebra, the '''Factor theorem''' is a theorem regarding the relationships between the factors of a polynomial a ...ath>f(a)</math> (<math>a</math> is constant, <math>f</math> is polynomial) is <math>0</math> using polynomial division rather than plugging in large valu
    3 KB (508 words) - 12:31, 2 May 2024
  • '''L'Hopital's Rule''' is a theorem dealing with [[limit]]s that is very important to [[calculus]]. The definition of a derivative is <math>f'(x) = \lim_{h\rightarrow 0} \frac{f(x+h)-f(x)}{h}</math> which can
    2 KB (475 words) - 15:04, 24 March 2022
  • A '''summation''' is the [[sum]] of a number of terms (addends). Summations are often written us ...index of summation, <math>a</math> is the lower bound, and <math>b</math> is the upper bound.
    3 KB (482 words) - 16:39, 8 October 2023
  • ...h>, where <math>\sigma</math> ranges over all permutations of <math>(1, 2, 3, \dots, n)</math>. ...generally, a '''symmetric sum''' of <math>n</math> variables is a sum that is unchanged by any [[permutation]] of its variables.
    1 KB (255 words) - 12:52, 8 October 2023
  • <math>5*1-1=4</math> ...the side lengths are all distinct, and that the side across from vertex C is 13 inches in length. How long are the other two sides?
    3 KB (458 words) - 15:44, 1 December 2015
  • '''Zorn's Lemma''' is a [[set theory | set theoretic]] result which is equivalent to the [[Axiom of Choice]]. ...per bound]], i.e., an upper bound <math>a</math> so that if <math>b</math> is an upper bound of <math>T</math>, then <math>a \le b</math>.
    9 KB (1,669 words) - 19:02, 1 August 2018
  • ''This article is about the median used in statistics. For other medians, check [[Median (dis A '''median''' is a measure of central tendency used frequently in statistics.
    2 KB (370 words) - 15:30, 12 November 2023
  • ...mbers and a partial [[inverse]] to [[exponentiation]]. (The proper inverse is the [[logarithm]]) A known method to compute all the roots of <math>\sqrt[n]{x}</math> is by the DeMoivre's formula.
    3 KB (532 words) - 16:52, 20 May 2020
  • ...s erased and placed in front of the remaining digits, the resulting number is four times as large as the original number <math>n</math>. <math>\sqrt{\sqrt{3-x}-\sqrt{x+1}}>\dfrac{1}{2}</math>
    3 KB (431 words) - 21:17, 20 August 2020
  • ...ge value of all the pennies, nickels, dimes, and quarters in Paula's purse is <math>20</math> cents. If she had one more quarter, the average value would ...t {(A)}\ 0 \qquad \text {(B)}\ 1 \qquad \text {(C)}\ 2 \qquad \text {(D)}\ 3\qquad \text {(E)}\ 4</math>
    2 KB (310 words) - 17:39, 31 December 2022
  • ...ct answer is worth <math>0</math> points, and each problem left unanswered is worth <math>2.5</math> points. If Charlyn leaves <math>8</math> of the <mat {{AMC12 box|year=2004|ab=A|num-b=1|num-a=3}}
    837 bytes (122 words) - 20:14, 3 July 2013
  • ...and <math>\overline{BB'}</math> intersect at <math>C = (2,8)</math>. What is the length of <math>\overline{A'B'}</math>? ...sqrt2 \qquad \text {(C)} 3 \qquad \text {(D)} 2 + \sqrt 2\qquad \text {(E)}3\sqrt 2</math>
    1 KB (181 words) - 20:17, 3 July 2013
  • *'''Case 1''': The line is horizontal or vertical, clearly <math>3 \cdot 2 = 6</math>. *'''Case 3''': The only remaining lines pass through two points, a vertex and a non-ve
    2 KB (381 words) - 00:58, 19 October 2020
  • ...d term, the three resulting numbers form a [[geometric progression]]. What is the smallest possible value for the third term in the geometric progression ...allest possible value occurs when <math>d = -14</math>, and the third term is <math>2(-14) + 29 = 1\Rightarrow\boxed{\mathrm{(A)}\ 1}</math>.
    4 KB (689 words) - 03:35, 16 January 2023
  • What is the value of <math>f(2^{100})</math>? f\left(2^3\right) &= 2^2\cdot f\left(2^2\right) &&= 2^{2+1}, \\
    2 KB (233 words) - 08:14, 6 September 2021
  • ...and <math>c</math>, respectively, are rounded to the nearest integer. What is the [[probability]] that <math>A + B = C</math>? ...=c=\frac 12</math>, and we want <math>c < \frac 12</math>. Thus the chance is <math>\frac{\frac{1}{4}}2 = \frac 18</math>.
    3 KB (552 words) - 23:26, 28 December 2020
  • ...a horizontal plane. A sphere of radius <math>2</math> rests on them. What is the distance from the plane to the top of the larger sphere?<!-- don't remo ...\frac {\sqrt {123}}{4}\qquad \text {(D)}\ \frac {52}{9}\qquad \text {(E)}\ 3 + 2\sqrt2</math>
    2 KB (382 words) - 19:59, 29 January 2023
  • ...th> are positive integers and <math>n</math> is as small as possible. What is <math>m</math>? ...<math>= \frac{x^2 + 3x + 3}{x^3 - 1}</math> <math>= \frac{(x+1)^3 - 1}{x(x^3 - 1)}</math>.
    3 KB (480 words) - 14:50, 17 August 2020
  • ...at <math>f(3x-1)=x^2+x+1</math> for all real numbers <math>x</math>. What is <math>f(5)</math>? <math>f(3(2)-1) = 2^2+2+1=7 \implies (A)</math>
    460 bytes (67 words) - 01:01, 19 October 2020
  • ...C}</math> and <math>\overline{BE}</math> intersect at <math>D</math>. What is the difference between the areas of <math>\triangle{ADE}</math> and <math>\ ...>[\triangle EBA]</math> is 16 and the area of <math>[\triangle ABC]</math> is 12. Set the area of <math>[\triangle ADB]</math> to be x. We want to find <
    5 KB (692 words) - 21:15, 5 April 2024
  • '''Binet's formula''' is an explicit formula used to find the <math>n</math>th term of the Fibonacci It is so named because it was derived by mathematician Jacques Philippe Marie Bin
    3 KB (550 words) - 16:12, 24 February 2024
  • pair O=(0,0), a=expi(0), b=expi(1/6), c=expi(2/6), d=expi(3/6), y=expi(32/30), z= expi(34/30); ...rc</math>. It then follows that the area of triangle <math>M_aOM_b</math> is <math>\tfrac{1}{2} \sin(b-a)^\circ \cdot OM_a \cdot OM_b = \tfrac{1}{2} \si
    4 KB (628 words) - 07:41, 19 July 2016
  • ...and <math>y \pmod{3}</math>, since <math>21</math> is a multiple of <math>3</math>. * Case 1: <math>x\equiv 0\pmod{3}</math>
    1 KB (250 words) - 00:38, 28 October 2015
  • ...<math>Q</math> is <math>20 \%</math> of <math>P</math>, and <math>N</math> is <math>50 \%</math> of <math>P</math>, then <math>\frac {M}{N} =</math> ...) \ 1 } \qquad \mathrm{(D) \ \frac {6}{5} } \qquad \mathrm{(E) \ \frac {4}{3} } </math>
    2 KB (232 words) - 22:30, 16 February 2018
  • </asy>|right|The de Longchamps <br />point (<math>L</math>) is the the <br />[[orthocenter]] (<math>H</math>) reflected <br /> through the The '''de Longchamps point''' of a [[triangle]] is the [[reflection (mathematics)|reflection]] of the triangle's [[orthocenter
    10 KB (1,780 words) - 09:23, 17 November 2022
  • Let <math>a_{1}, a_{2}, \dots, a_{n}</math> (<math>n > 3</math>) be real numbers such that Since <math>-a_i</math> is a positive real for all <math>k+1 \le i \le n</math>, it follows that
    3 KB (499 words) - 09:47, 20 July 2016
  • The answer is <math>C=1/8</math>, and equality holds exactly when two of the <math>x_i</m \sum_{i=1}^{k-1} x_i^3 (x_k + x_{k+1}) + S(x_k^3 + x_{k+1}^3) \\
    4 KB (838 words) - 01:04, 17 November 2023
  • ...me proofs of the weighted [[AM-GM Inequality]]. The inequality's statement is as follows: for all nonnegative reals <math>a_1, \dotsc, a_n</math> and non ...ht-hand side of the inequality is zero and the left hand of the inequality is greater or equal to zero, with equality if and only if <math>a_j = 0 = a_i<
    12 KB (2,171 words) - 07:55, 11 May 2023
  • <cmath> P(x^5) + xQ(x^5) + x^2 R(x^5) = (x^4 + x^3 + x^2 + x +1) S(x), </cmath> prove that <math>x-1</math> is a factor of <math>P(x)</math>.
    3 KB (572 words) - 17:14, 16 August 2015
  • ...tism]]), but it is the only one which is always attractive, which means it is much more noticeable than the others. ...rmulas so that they covered slight discrepancies with actual results. This is the theory generally accepted today.
    2 KB (318 words) - 17:36, 10 March 2014
  • ...condition: for every [[natural number]] <math>n</math>, <math>2^n-1</math> is divisible by <math>W(n)</math>. ...on "natural number" is ambiguous, but this is irrelevant, as every integer is a divisor of <math>2^0 -1 = 0</math>.''
    5 KB (919 words) - 23:29, 20 January 2016
  • The first number is divisible by <math>1</math>. The sum of the first two numbers is even.
    4 KB (601 words) - 14:27, 19 February 2024
  • ...<math>x</math>-axis to produce <math>R</math>, and finally, <math>R</math> is translated by 5 units in the positive-<math>y</math> direction to produce < ...C) } ( - 1, - 2,8) \qquad \text {(D) } ( - 1,3,3) \qquad \text {(E) } (1,3,3)
    1 KB (173 words) - 08:56, 21 August 2023
  • ...logarithms of the divisors of <math>10^n</math> is <math>792</math>. What is <math>n</math>? ...>2^{n-a} \times 5^{n-b}</math>. Note this is not true if <math>10^n</math> is a perfect square. When these are multiplied, they equal <math>2^{a+n-a} \ti
    5 KB (814 words) - 18:02, 17 January 2023
  • ...nd <math>b,</math> satisfy <math>ab = a - b</math>. Which of the following is a possible value of <math>\frac {a}{b} + \frac {b}{a} - ab</math>? ...- 2 \qquad \textbf{(B)} \ \frac { -1 }{2} \qquad \textbf{(C)} \ \frac {1}{3} \qquad \textbf{(D)} \ \frac {1}{2} \qquad \textbf{(E)} \ 2</math>
    2 KB (405 words) - 21:48, 21 April 2024
  • In [[graph theory]], a '''graph''' is a (usually [[finite]]) [[empty set | nonempty]] [[set]] of [[vertex|vertice ...>. Note that this definition describes ''simple, loopless'' graphs: there is at most one edge joining two vertices, no edge may join a vertex to itself,
    8 KB (1,428 words) - 10:26, 27 August 2020
  • <math>\text {(A)}\ 3 \qquad \text {(B)}\ 4 \qquad \text {(C)}\ 5 \qquad \text {(D)}\ 6 \qquad \t ...ollows that <math>6-p</math> and <math>p-4</math> are also positive, which is only possible when <math>p = 5\ \mathrm{(C)}</math>.
    6 KB (996 words) - 18:14, 29 June 2023
  • Let <math>f</math> be a function for which <math>f\left(\dfrac{x}{3}\right) = x^2 + x + 1</math>. Find the sum of all values of <math>z</math> ...\ -1/9 \qquad \text {(C)}\ 0 \qquad \text {(D)}\ 5/9 \qquad \text {(E)}\ 5/3</cmath>
    3 KB (441 words) - 21:11, 29 April 2023
  • ...board. If the board is renumbered so that the left column, top to bottom, is <math>1,2,\ldots,13,</math>, the second column <math>14,15,\ldots,26</math> Index the rows with <math>i = 1, 2, 3, ..., 13</math>
    2 KB (310 words) - 11:28, 3 August 2021
  • ...</math> and contains the point <math>A</math>. The segment <math>AB</math> is tangent to the circle at <math>A</math> and <math>\angle AOB = \theta</math dotfactor=3;
    6 KB (979 words) - 12:50, 17 July 2022
  • ...ngle bisector|bisector]] of angle <math>BAC</math>. Which of the following is closest to the area of the triangle <math>ADE</math>? ...{(A)}\ 2 \qquad \text {(B)}\ 2.5 \qquad \text {(C)}\ 3 \qquad \text {(D)}\ 3.5 \qquad \text {(E)}\ 4</math>
    3 KB (547 words) - 17:37, 17 February 2024
  • ...d y + \frac{1}{z} = 1, \qquad \text{and} \qquad z + \frac{1}{x} = \frac{7}{3}</cmath> Then what is the value of <math>xyz</math> ?
    2 KB (244 words) - 07:34, 4 November 2022
  • ...]] of the area of the other small right triangle to the area of the square is draw((0,0)--(6,0)--(0,3)--cycle);
    5 KB (804 words) - 01:22, 13 May 2024
  • ...l]] <math>P(x) = x^4 + ax^3 + bx^2 + cx + d</math>. Which of the following is the smallest? Note that there are 3 maxima/minima. Hence we know that the rest of the graph is greater than 10. We approximate each of the above expressions:
    1 KB (212 words) - 14:37, 5 June 2022
  • ...he same property— the sum of the base-ten logarithms is an integer. What is the [[probability]] that Professor Gamble holds the winning ticket? <math>\textbf {(A)}\ 1/5 \qquad \textbf {(B)}\ 1/4 \qquad \textbf {(C)}\ 1/3 \qquad \textbf {(D)}\ 1/2 \qquad \textbf {(E)}\ 1</math>
    2 KB (348 words) - 22:26, 24 October 2021
  • ...verarc{BC}</math> is <math>12</math>, then the circumference of the circle is label("C", (32, 32*sqrt(3)), N);
    2 KB (263 words) - 19:59, 18 April 2024
  • Since the octahedron is indistinguishable by rotations, without loss of generality fix a face to be ...e three possible rotations about the fixed face, so the answer is <math>7!/3 = 1680</math>.
    7 KB (998 words) - 21:04, 23 December 2023
  • <math>\text{(A)}\ 3 \qquad \text{(B)}\ 4 \qquad \text{(C)}\ 6 \qquad \text{(D)}\ 9 \qquad \text <cmath>m^2 - n^2 = (m+n)(m-n) = 96 = 2^{5} \cdot 3</cmath>
    2 KB (352 words) - 14:20, 3 July 2023
  • ...<math>n</math> digits, each of which is 8. Prove that <math>A+2B+4</math> is a perfect square. &= \left(\frac{2(10^n + 2)}{3}\right)^2.
    1,002 bytes (145 words) - 00:14, 28 August 2018
  • ...<math>a</math> satisfies the equation <math>4 = a + a^{ - 1}</math>. What is the value of <math>a^{4} + a^{ - 4}</math>? == Solution 3 (Detailed Explanation of Solution 2) ==
    4 KB (663 words) - 07:32, 4 November 2022
  • ...<math>a@b = ab - b^{2}</math> and <math>a\#b = a + b - ab^{2}</math>. What is <math>\frac {6@2}{6\#2}</math>? == Problem 3 ==
    13 KB (2,058 words) - 17:54, 29 March 2024
  • ...th> square meters. A second cube is then inscribed within the sphere. What is the surface area in square meters of the inner cube? <math>\text{(A)}\ 3 \qquad \text{(B)}\ 6 \qquad \text{(C)}\ 8 \qquad \text{(D)}\ 9 \qquad \text
    4 KB (609 words) - 14:41, 3 December 2023
  • ...act the <math>2</math> cases where a guide has no tourist. Thus the answer is <math>2^6 - 2 = \boxed{62}\ \mathrm{(D)}</math>. ...ts go to tour guide A. Thus, we can see that the total number of groupings is:
    3 KB (411 words) - 18:07, 14 March 2023
  • ...h a manner that the sum of the four numbers on each face is the same. What is this common sum? ...</math> numbers represent all of the vertices of the cube. Thus the answer is <math>\frac{1 + 2 + \cdots + 8}{2} = 18\ \mathrm{(C)}</math>.
    1 KB (195 words) - 20:44, 21 January 2024
  • ...<math>48</math> years old, and the average age of the mother and children is <math>16</math>. How many children are in the family? <math>\text{(A)}\ 2 \qquad \text{(B)}\ 3 \qquad \text{(C)}\ 4 \qquad \text{(D)}\ 5 \qquad \text{(E)}\ 6</math>
    2 KB (411 words) - 20:54, 19 August 2023
  • ...<math>3^{a} = 81^{b + 2}</math> and <math>125^{b} = 5^{a - 3}</math>. What is <math>ab</math>? <cmath>81^{b+2} = 3^{4(b+2)} = 3^a \Longrightarrow a = 4b+8</cmath>
    2 KB (268 words) - 12:40, 3 June 2021
  • The larger of two consecutive odd integers is three times the smaller. What is their sum? ...+2</math>. Then <math>a+2 = 3a</math>, so <math>a = 1</math> and their sum is <math>4\ \mathrm{(A)}</math>.
    525 bytes (79 words) - 00:58, 28 April 2021
  • ...<math>a@b = ab - b^{2}</math> and <math>a\#b = a + b - ab^{2}</math>. What is <math>\frac {6@2}{6\#2}</math>? ...equal to <math>\frac{8}{-16} = -\frac{1}{2}</math>. Therefore the solution is <math>A</math>.
    730 bytes (108 words) - 15:07, 27 January 2021
  • ...mathrm{(C) \ \text{increase by 2} } \qquad \mathrm{(D) \ \text{increase by 3} } \qquad \mathrm{(E) \ \text{increase by 4} } </math> If <math>\sqrt {2 + \sqrt {x}} = 3</math>, then <math>x =</math>
    17 KB (2,387 words) - 22:44, 26 May 2021
  • What is <math>A + B + C</math>? ...</math> and <math>C</math> are relatively prime, <math>C=1</math>, <math>B=3</math>, <math>A=2</math>. <math>A+B+C=6 \Rightarrow \mathrm{(A)}</math>
    977 bytes (142 words) - 14:05, 5 July 2013
  • ...dway between them is of length <math>\sqrt {a}</math> where <math>a</math> is ...f the other leg. Now the length of the leg of <math>\triangle OAA_1</math> is either <math>6 + x</math> or <math>6 - x</math> depending whether or not <m
    3 KB (509 words) - 22:56, 5 December 2023
  • ...th>\{1,2,3,4,5 \}</math>, the probability that <math>ab + c</math> is even is ...1}{2} } \qquad \mathrm{(D) \ \frac {64}{125} } \qquad \mathrm{(E) \ \frac {3}{5} } </math>
    1 KB (177 words) - 18:05, 29 July 2022
  • ...mathrm{(C) \ \text{increase by 2} } \qquad \mathrm{(D) \ \text{increase by 3} } \qquad \mathrm{(E) \ \text{increase by 4} } </math> ...cores is <math>\frac{88+83+87}{3}=86</math>. The average of all four exams is <math>\frac{87+83+88+90}{4}=87</math>. It increased by one point. <math>\ma
    701 bytes (101 words) - 13:58, 5 July 2013
  • ..., f(3) \geq f(4),</math> and <math>f(5) = 5</math>. Which of the following is true? ...if <math>f(a)=f(b)</math> for <math>a\neq b</math>, then <math>f(x)</math> is a constant function. Since <math>f(5)=5</math>, <math>f(0)=5\Rightarrow \ma
    2 KB (353 words) - 13:26, 4 May 2022
  • ...e]]s of the other two vertices are integers. The number of such rectangles is <math> \mathrm{(A) \ 1 } \qquad \mathrm{(B) \ 2 } \qquad \mathrm{(C) \ 3 } \qquad \mathrm{(D) \ 4 } \qquad \mathrm{(E) \ 5 } </math>
    1 KB (187 words) - 18:30, 4 December 2020
  • ...ny three-element sets of distinct positive integers <math>\{a,b,c\}</math> is it true that <math>a \times b \times c = 2310</math>? ...to figure out the number of ways to distribute these prime factors amongst 3 different integers, without over counting triples which are simply permutat
    2 KB (371 words) - 14:33, 15 January 2023
  • Peter is ill. He has to take medicine <math>A</math> every <math>8</math> hours, med ...)--(1.45,0.54)--(1.38,0.46)--(1.31,0.46)--(1.35,0.38)--(1.31,0.3)--(1.38,0.3)--cycle);
    15 KB (2,057 words) - 19:13, 10 March 2015
  • ...the number that most closely approximates the number of ants in the field is The rectangular field is <math>300 \text{ feet} \cdot \frac{12 \text{ inches}}{1 \text{ foot}} = 36
    1 KB (154 words) - 13:58, 5 July 2013
  • ...x) = ax^4 - bx^2 + x + 5</math> and <math>f( - 3) = 2</math>, then <math>f(3) =</math> ...} \qquad \mathrm{(B) \ -2 } \qquad \mathrm{(C) \ 1 } \qquad \mathrm{(D) \ 3 } \qquad \mathrm{(E) \ 8 } </math>
    1 KB (202 words) - 18:05, 6 April 2016
  • ...]] bounded by the lines <math>y = x, y = - x</math> and <math>y = 6</math> is label("$y=x$",(3,3),(1,-1));label("$y=-x$",(-3,3),(-1,-0.5));
    956 bytes (126 words) - 13:59, 5 July 2013
  • ...eometry]] and was compiled by [[Euclid]] in the time of ancient Greece. It is divided into thirteen volumes, each consisting of definitions, "common noti ...sibly translated into Latin during the reign of the Roman Empire, but this is doubtful. The Arabs acquired copies of ''The Elements'' circa 750 AD, and i
    12 KB (2,094 words) - 15:42, 1 December 2015
  • The percent that <math>M</math> is greater than <math>N</math> is: ...ed by <math>x</math> yards of fencing. The area in terms of <math>x</math> is:
    23 KB (3,641 words) - 22:23, 3 November 2023
  • ...ed by <math>x</math> yards of fencing. The area in terms of <math>x</math> is: ...2w^2 = 2\left(\frac{x^2}{36}\right) = \frac{x^2}{18}</math>, so the answer is <math>\mathrm{D}</math>.
    743 bytes (121 words) - 12:19, 5 July 2013
  • 19. Prove that for any odd integer <math>n \geq 1</math>, there is a way to number so that each integer is used once, and adjacent beads correspond to
    3 KB (460 words) - 19:14, 18 July 2016
  • ...h> is a <math>k</math>-good sequence which starts on <math>a</math>, so it is a permutation of <math>k</math> consecutive integers, say <math>m, \dotsc, ...<math>k</math> digits. Thus the total number of desired positive integers is
    3 KB (529 words) - 19:15, 18 July 2016
  • ...plication is a common method of solving [[proportions]]. Essentially, one is multiplying both sides by the denominators of both sides. \frac{x}{3} &= \frac{10}{5} \\
    4 KB (562 words) - 18:49, 8 November 2020
  • ...<math>50</math>%. The percent of increase of the surface area of the cube is: Through a point <math>P</math> inside the <math>\triangle ABC</math> a line is drawn parallel to the base <math>AB</math>, dividing the triangle into two
    22 KB (3,345 words) - 20:12, 15 February 2023
  • ...o of <math>3x - 4</math> to <math>y + 15</math> is constant, and <math>y = 3</math> when <math>x = 2</math>, then, when <math>y = 12</math>, <math>x</ma ...sed 10% and the altitude to this base is decreased 10%, the change in area is
    19 KB (3,159 words) - 22:10, 11 March 2024
  • ...>1</math> and their product is <math>1</math>, then the sum of their cubes is: ...i\sqrt {3}}{4} \qquad \text{(C)} \ 0 \qquad \text{(D)} \ - \frac {3i\sqrt {3}}{4} \qquad \text{(E)} \ - 2</math>
    611 bytes (97 words) - 12:40, 5 July 2013
  • ...] of <math>3x - 4</math> to <math>y + 15</math> is constant, and <math>y = 3</math> when <math>x = 2</math>, then, when <math>y = 12</math>, <math>x</ma ...frac{1}{9} = \frac{3x - 4}{(12) + 15}</math>. Solving gives <math>3x - 4 = 3 \Longrightarrow x = \frac 73 \Rightarrow \mathrm{(B)}</math>.
    712 bytes (99 words) - 12:39, 5 July 2013
  • is eventually constant. [The tower of exponents is defined by <math>a_1 = 2, \; a_{i+1} = 2^{a_i}</math>. Also <math>a_i \pmod
    3 KB (559 words) - 19:18, 5 June 2023
  • ..., angle <math>A</math> is twice angle <math>B</math>, angle <math>C</math> is [[obtuse triangle|obtuse]], and the three side lengths <math>a, b, c</math> //y = 4x/3 and x+2y = 2 (sides AC and BC, respectively) intersect here:
    5 KB (755 words) - 08:58, 6 May 2023
  • ...\frac{\sigma(S)}{\pi(S)} = (n^2 + 2n) - \left( 1 + \frac{1}{2} + \frac{1}{3} + \cdots + \frac{1}{n} \right) (n+1), </cmath> ...denotes a sum involving all nonempty subsets <math>S</math> of <math>\{1,2,3, \ldots,n\}</math>.
    3 KB (490 words) - 07:38, 19 July 2016
  • ...h>\,A\,</math> is twice angle <math>\,B,\,</math> angle <math>\,C\,</math> is obtuse, and the three side lengths <math>\,a,b,c\,</math> are integers. De ...\frac{\sigma(S)}{\pi(S)} = (n^2 + 2n) - \left( 1 + \frac{1}{2} + \frac{1}{3} + \cdots + \frac{1}{n} \right) (n+1), </cmath>
    3 KB (512 words) - 19:17, 18 July 2016
  • == Solution 3 (Solution 1 but shorter) == ...her variable to get <math>15x^2=960</math> and <math>x^2=64</math>. Now XY is equal to the square root of four times these quantities, so <math>(105+64)
    3 KB (447 words) - 15:02, 17 August 2023
  • The area of <math>R</math> is closest to <cmath>x^2 + 6x + 1 + y^2 + 6y + 1 \le 0 \Longrightarrow (x+3)^2 + (y+3)^2 \le 16</cmath>
    2 KB (365 words) - 14:48, 7 March 2022
  • ...om <math>A</math> to <math>\overline{BC}</math> have the same length. What is <math>BC</math>? \qquad\mathrm{(B)}\ \frac{1+\sqrt{3}}2
    4 KB (598 words) - 09:45, 23 January 2024
  • <cmath>\begin{align*}b- c &= \left(\frac{1}{\log 2002}\right)(\log 2 + \log 3 + \log 4 + \log 5 - \log 10 - \log 11 - \log 12 - \log 13 - \log 14)\\ &= \left(\frac{1}{\log 2002}\right)\left(\log \frac{2 \cdot 3 \cdot 4 \cdot 5}{10 \cdot 11 \cdot 12 \cdot 13 \cdot 14}\right)\\
    2 KB (312 words) - 18:26, 13 January 2022
  • ...is the [[probability]] that the product of the two rolls is a multiple of 3? ...frac{2}{8} = \frac{1}{4}</math> be the probability that Juan rolls a <math>3</math> or a <math>6</math>, and <math>P(b) = \frac{2}{6} = \frac 13</math>
    2 KB (317 words) - 10:26, 5 November 2023
  • ...ge--><onlyinclude>Four distinct [[circle]]s are drawn in a [[plane]]. What is the maximum number of points where at least two of the circles intersect?<! ...t 2 = 12</math>. We can construct such a situation as below, so the answer is <math>\boxed{\mathrm{(D)}\ 12}</math>.
    2 KB (282 words) - 14:04, 12 July 2021
  • ...<math>P</math> is closer to the [[origin]] than it is to the point <math>(3,1)</math>? ...le is <math>2 - \frac 12 = \frac 32</math>. The probability is <math>\frac{3/2}{2} = \frac 34 \Rightarrow \mathrm{(C)}</math>.
    3 KB (376 words) - 19:16, 20 August 2019
  • ...> and <math>A+B</math> are all prime numbers. The sum of these four primes is \qquad\mathrm{(B)}\ \mathrm{divisible\ by\ }3
    3 KB (470 words) - 11:57, 10 August 2022
  • For how many integers <math>n</math> is <math>\dfrac n{20-n}</math> the [[perfect square|square]] of an integer? \qquad\mathrm{(C)}\ 3
    4 KB (579 words) - 05:54, 17 October 2023
  • ...C 12B Problems|2002 AMC 12B #1]] and [[2002 AMC 10B Problems|2002 AMC 10B #3]]}} ...ne numbers in the set <math>\{9, 99, 999, 9999, \ldots, 999999999\}</math> is a <math>9</math>-digit number <math>M</math>, all of whose digits are disti
    1 KB (185 words) - 19:54, 23 June 2023
  • What is the value of <math>(3x - 2)(4x + 1) - (3x - 2)4x + 1</math> when <math>x=4 <cmath>(3x-2)[(4x+1)-4x] + 1 = 3x-2 + 1 = 3x-1 = 3(4) - 1 = \boxed{\mathrm{(D)}\ 11}</cmath>.
    802 bytes (105 words) - 10:46, 9 August 2022
  • {{duplicate|[[2002 AMC 12B Problems|2002 AMC 12B #3]] and [[2002 AMC 10B Problems|2002 AMC 10B #6]]}} For how many positive integers <math>n</math> is <math>n^2 - 3n + 2</math> a [[prime]] number?
    1 KB (247 words) - 21:30, 24 August 2022
  • ...(as a pentagon can be split into <math>5- 2 = 3</math> triangles) is <math>3 \cdot 180 = 540^{\circ}</math>. If we let <math>v = x - 2d, w = x - d, y = ...the middle term of an arithmetic sequence with an odd number of terms, it is simply the average of the sequence.
    2 KB (263 words) - 05:58, 8 August 2023
  • ...frac 17 + \frac 1n</math> is an integer. Which of the following statements is '''not ''' true: \qquad\mathrm{(B)}\ 3\ \text{divides\ }n
    2 KB (321 words) - 10:54, 9 August 2022
  • ...utions <math>a</math> and <math>b</math>. Then the pair <math>(a,b)</math> is ...<math>-a - b = a</math> and that <math>ab = b</math>. Since <math>b</math> is nonzero, <math>a = 1</math>, and <math>-1 - b = 1 \Longrightarrow b = -2</m
    3 KB (457 words) - 14:53, 17 August 2023
  • ...gers with three distinct digits. Compute the remainder when <math>S</math> is divided by <math>1000</math>. ...e tens and units digits. Thus the sum of the hundreds places is <math>(1+2+3+\cdots+9)(72) \times 100 = 45 \cdot 72 \cdot 100 = 324000</math>.
    1 KB (194 words) - 13:44, 5 September 2012
  • ...nd externally tangent to a circle of radius <math>2</math>, as shown. What is the area of the square? real h=3*sqrt(2)/2;
    2 KB (326 words) - 10:29, 4 June 2021
  • ...h>\cos 20^{\circ}</math> is <math>8x^3-6x-1</math>, which has degree <math>3</math>. ...of <math>\sqrt[3]{2}</math> is <math>x^3-2</math>, which has degree <math>3</math>.
    2 KB (359 words) - 17:06, 11 June 2022
  • A '''compass''' is a tool that can draw circles and arcs of circles. A '''straightedge''' is an unmarked ruler that can draw line segments.
    3 KB (443 words) - 20:52, 28 August 2014
  • ...spaghetti. The results of the survey are displayed in the bar graph. What is the ratio of the number of students who preferred spaghetti to the number o ...} \frac{1}{2} \qquad \textbf{(C)} \frac{5}{4} \qquad \textbf{(D)} \frac{5}{3} \qquad \textbf{(E)} \frac{5}{2}</math>
    12 KB (1,800 words) - 20:01, 8 May 2023
  • ...values are there of <math>k</math>, where <math>k</math> is at least <math>3</math> and at most <math>2008</math>? ...that student can either stay still, for which the number of rearrangements is simply the number of rearrangements for the other <math>n</math> students (
    3 KB (497 words) - 13:29, 20 October 2019
  • The mean of three numbers is <math>10</math> more than the least of the numbers and <math>15</math> ...than the greatest. The median of the three numbers is <math>5</math>. What is their
    2 KB (311 words) - 21:53, 10 February 2024
  • Since there are five types of squares: <math>1 \times 1, 2 \times 2, 3 \times 3, 4 \times 4,</math> and <math>5 \times 5.</math> We must find how many of e *There is <math>1</math> of all <math>1\times 1</math> squares, containing the black
    2 KB (339 words) - 00:25, 14 February 2024
  • ...ates: <math>(1,8,15,22,29)</math>, <math>(2,9,16,23,30)</math>, and <math>(3,10,17,24,31)</math>. The only day of the week that is guaranteed to appear five times is therefore <math>\boxed{\textrm{(D)}\ \text{Thursday}}</math>.
    2 KB (329 words) - 15:48, 14 October 2023
  • ...d <math>a,b,d</math> form a geometric sequence, then <math>\frac ad</math> is We can let <math>a=1</math>, <math>b=2</math>, <math>c=3</math>, and <math>d=4</math>. <math>\frac{a}{d}=\boxed{\frac{1}{4}} \Longr
    2 KB (288 words) - 21:42, 11 December 2017
  • ...math> and <math>\overline{BD}</math>, respectively. Which of the following is closest to the minimum possible value of <math>PQ</math>? ...sector]]s. Thus <math>\angle POQ = 90^{\circ}</math>, so <math>OPNQ</math> is a [[rectangle]]. Since the diagonals of a rectangle are of equal length, <m
    4 KB (551 words) - 14:17, 23 June 2022
  • ...h>, and moves <math>5</math> cm in a straight line to <math>C</math>. What is the [[probability]] that <math>AC < 7</math>? \qquad\mathrm{(D)}\ \frac{1}{3}
    2 KB (372 words) - 09:38, 1 September 2022
  • ...sults by 3, we obtain the set <math>\{-3, -2, -1, 0, 1, 2, 3\}</math>. It is easy to see that we can get any integer between <math>-6</math> and <math>6 ...e numbers will be a multiple of <math>3</math>. All the multiples of <math>3</math> from <math>1+4+7=12</math> to <math>13+16+19=48</math> are possible,
    1 KB (166 words) - 00:43, 17 January 2021
  • ...ine{AG}</math> and <math>\overline{CH}</math> meet at <math>M</math>. What is the area of quadrilateral <math>ABCM</math>? pair A=(1,3), B=(2,3), C=(2,2), D=(3,2), Ep=(3,1), F=(2,1), G=(2,0), H=(1,0), I=(1,1), J=(0,1), K=(0,2), L=(1,2);
    6 KB (867 words) - 00:17, 20 May 2023
  • Part of the graph of <math>f(x) = ax^3 + bx^2 + cx + d</math> is shown. What is <math>b</math>? <cmath>a(x-1)(x+1)(x-n) = ax^3-anx^2-ax+an = ax^3 + bx^2 + cx + d = 0</cmath>
    2 KB (382 words) - 18:01, 23 November 2020
  • If <math>\log (xy^3) = 1</math> and <math>\log (x^2y) = 1</math>, what is <math>\log (xy)</math>? <cmath>3\log(xy) + 2\log y + 2\log x = 3 \Longrightarrow 5\log(xy) = 3</cmath>
    2 KB (329 words) - 13:49, 4 April 2024
  • Which of the following is the same as <cmath>\frac{2-4+6-8+10-12+14}{3-6+9-12+15-18+21}?</cmath>
    970 bytes (134 words) - 00:09, 14 September 2015
  • ...ted on the silo, as shown, making two complete revolutions around it. What is the area of the stripe in square feet? draw(ellipse(origin, 3, 1));
    2 KB (220 words) - 14:19, 21 April 2021
  • ...vely, and <math>EF</math> is a common [[tangent (geometry)|tangent]]. What is the area of the shaded region <math>ECODF</math>? <math>\text{(A)}\ \frac {8\sqrt {2}}{3} \qquad \text{(B)}\ 8\sqrt {2} - 4 - \pi \qquad \text{(C)}\ 4\sqrt {2} \qqu
    3 KB (439 words) - 15:39, 3 June 2021
  • ...etric painted area, as shown. Half the area of the square is painted. What is the ratio of the side length of the square to the brush width? unitsize(3 cm);
    4 KB (560 words) - 14:57, 3 June 2021
  • ...second term is <math>2</math>, in lowest terms, is <math>a/b</math>. What is <math>a+b</math>? ...the answer is <math>\frac{3 \cdot 3!}{4 \cdot 4!} = \frac {18}{96} = \frac{3}{16}</math>, and <math>a+b=19 \Rightarrow \mathrm{(E)}</math>.
    2 KB (320 words) - 22:59, 5 May 2024
  • has exactly one solution. What is the minimum value of <math>c</math>? When <math>x<a</math>, the slope is <math>-3</math>.
    7 KB (1,183 words) - 11:47, 15 February 2016
  • ...eam occupies <math>75\%</math> of the volume of the frozen ice cream. What is the ratio of the cone’s height to its [[radius]]? \qquad\mathrm{(B)}\ 3:1
    960 bytes (154 words) - 15:20, 17 October 2016
  • What is the value of the [[expression]] Thus the sum is
    1 KB (138 words) - 10:41, 22 April 2016
  • In [[quadrilateral]] <math>ABCD</math>, it is given that <math>\angle A = 120^{\circ}</math>, angles <math>B</math> and < ...ath>\triangle ABE \sim \triangle CDE</math>, so <math>\triangle CDE</math> is also a <math>30-60-90</math> triangle.
    3 KB (391 words) - 14:30, 5 July 2013
  • .../math> denote a polynomial having all integral coefficients. Show that it is impossible that <math>P(a)=b</math>, <math>P(b)=c</math>, and <math>P(c)=a< If <math>P</math> is a polynomial with integral coefficients, then <cmath>a - b | P(a) - P(b).</
    7 KB (1,291 words) - 20:30, 27 April 2020
  • ...as each center cube are removed. The [[surface area]] of the final figure is: ...th>. Thus the total surface area for each <math>3 \times 3 \times 3</math> is <math>72</math>.
    3 KB (403 words) - 23:00, 10 August 2020
  • ...ng terms. Prove that, for all <math>\, n, \,</math> <math>\, a_n \,</math> is the number obtained by writing <math>\, n \,</math> in base <math>\, p-1 \, A calculator is broken so that the only keys that still work are the <math>\, \sin, \; \cos
    3 KB (540 words) - 13:31, 4 July 2013
  • ...h>2^{2004}</math> is a <math>604</math>-[[digit]] number whose first digit is <math>1</math>, how many [[element]]s of the [[set]] <math>S = \{2^0,2^1,2^ ...,9\}</math> [[iff|if and only if]] the first digit of <math>2^{k-1}</math> is <math>4</math>, so there are <math>\boxed{195} \Rightarrow \mathrm{(B)}</ma
    3 KB (383 words) - 20:12, 15 October 2016
  • ...three distinct positive zeros. Exactly one of these is an integer, and it is the sum of the other two. How many values of <math>n</math> are possible? <center><math>(x - r)(x - s)(x - (r + s))</math> <math>= x^3 - (r + s + r + s) x^2 + (rs + r(r + s) + s(r + s))x - rs(r + s) = 0</math><
    3 KB (533 words) - 14:52, 29 October 2023
  • ...ath> <math>\cot \angle DBC</math> form an [[arithmetic progression]]. What is the area of <math>\triangle ABC</math>? ..., D = (5*2^.5/3,0), C = (10*2^.5/3,0), B = (5*2^.5/3,5*2^.5), E = (13*2^.5/3,0);
    2 KB (302 words) - 19:59, 3 July 2013
  • ...angent to the top, bottom, and lateral surface of the truncated cone. What is the radius of the sphere? \qquad\mathrm{(E)}\ 6\sqrt{3}</math>
    3 KB (520 words) - 19:12, 20 November 2023
  • An acute-angled triangle <math>ABC</math> is given in the plane. The circle with diameter <math>\, AB \,</math> intersec ...ath>, <math>H</math>, <math>A'</math> are concurrent, where <math>H</math> is the orthocentre.
    3 KB (604 words) - 20:52, 24 October 2018
  • A calculator is broken so that the only keys that still work are the <math>\, \sin, \; \cos ...th>n=1</math> and <math>m=0</math>, and <math>\sqrt{m/n} = 0</math>, which is initially shown on the screen. For the inductive step, we consider separat
    3 KB (516 words) - 00:18, 6 April 2020
  • ...th>\frac yx</math> over all points <math>(x,y)</math> on the ellipse. What is the value of <math>a+b</math>? <math>\mathrm{(A)}\ 3
    2 KB (256 words) - 16:23, 13 March 2023
  • ...>f^{-1}(x) = bx+a</math> with <math>a</math> and <math>b</math> real, what is the value of <math>a+b</math>? ...ath>f(f^{-1}(x))=-(-x-1)-1=x+1-1=x.</cmath><cmath>f^{-1}(f(x))=-(-x-1)-1=x+1-1=x.</cmath>Therefore <math>a+b=\boxed{-2}</math>.
    1 KB (193 words) - 13:56, 2 March 2021
  • ...o sides of length <math>3</math> and a base of length <math>2</math>. What is the area of this circle? ...} \frac{5}{2}\pi \qquad\textbf{(C) } \frac{81}{32}\pi \qquad\textbf{(D) } 3\pi \qquad\textbf{(E) } \frac{7}{2}\pi</math>
    5 KB (851 words) - 22:02, 26 July 2021
  • ...let <math>\, s_m = k_1 + k_2 + \cdots + k_m \,</math> for <math>\, m = 1,2,3, \ldots \; \;</math>. Prove that, for each positive integer <math>\, n, \, ...t sides may be the same color. By making a sequence of such modifications, is it possible to arrive at the coloring in which consecutive sides
    2 KB (391 words) - 07:58, 19 July 2016
  • ...ath>CD = 39</math>, and <math>DA = 5</math>. The area of <math>ABCD</math> is ...e hard to use [[trigonometry]] to bash this and find the height, but there is a much easier way. Extend <math>\overline{AD}</math> and <math>\overline{BC
    8 KB (1,308 words) - 07:05, 19 December 2022
  • A set of tiles numbered 1 through 100 is modified repeatedly by the following operation: remove all tiles numbered w The pattern is quite simple to see after listing a couple of terms.
    3 KB (430 words) - 23:13, 13 September 2023
  • Sometimes it is convenient to take one pair of isogonals as the base one, for example, <mat ...h> into a point with coordinates <math>(\frac{1}{x}, \frac {y}{x}),</math> is projective.
    54 KB (9,416 words) - 08:40, 18 April 2024
  • ...qquad\mathrm{(B)}\ \text{3:00}\ {\small\text{PM}}\qquad\mathrm{(C)}\ \text{3:30}\ {\small\text{PM}}\qquad\mathrm{(D)}\ \text{4:30}\ {\small\text{PM}}\qq ...gth to its width is <math>2:1</math>. What percent of the rectangle's area is in the square?
    14 KB (2,138 words) - 15:08, 18 February 2023
  • Clearly, <math>P_1(x)</math> has 2 real solutions, where 1 is positive and 1 is negative. The absolute values of these two solutions are also both less tha ...have that <math>-2<s<2</math>, so <math>0<r^2<4</math>, so <math>r</math> is real and <math>|r|<2</math>. Therefore all of the roots of <math>P_{k+1}</m
    3 KB (596 words) - 16:19, 28 July 2015
  • ...f the box, then exactly <math>40</math> percent from the volume of the box is occupied. Determine the possible dimensions of the box. x &= \left\lfloor\frac{a}{\sqrt[3]{2}}\right\rfloor \\
    4 KB (605 words) - 16:27, 29 January 2021
  • ...who is the product of some positive integers, and the sum of these numbers is <math>1976.</math> ...>3*3=2*2*2+1</math>, 3's are more efficient than 2's. We try to prove that 3's are more efficient than anything:
    4 KB (668 words) - 04:29, 2 January 2023
  • A sequence <math>(u_{n})</math> is defined by <cmath>\lfloor u_{n} \rfloor = 2^{\frac {(2^{n} - ( - 1)^{n})}{3}}</cmath>
    3 KB (531 words) - 16:30, 29 January 2021
  • ...ee partitions of 3: <math>3 = 2+1 =1+1+1</math>. Each of the [[summand]]s is a ''part'' of the partition. ...formula! No simpler formula is known, and the existence of such a formula is doubtful.
    10 KB (1,508 words) - 14:24, 17 September 2017
  • What is the product of all positive odd integers less than <math>10000</math>? ...dot 4 \cdot 6 \cdots 10000}= \dfrac{10000!}{2^{5000} \cdot 1 \cdot 2 \cdot 3 \cdots 5000}= \dfrac{10000!}{2^{5000}\cdot5000!}</math>
    2 KB (226 words) - 00:30, 30 December 2023
  • order; that is, <math>A > B > C</math>, <math>D > E > F</math>, and <math>G > H > I > J</m ...1, 2, 3, 4, 5, 6, 7, 8, 9</math>. Clearly, the most restrictive condition is the consecutive odd digits, so we create casework based on that.
    2 KB (340 words) - 03:02, 28 June 2023
  • ...qquad\mathrm{(B)}\ \text{3:00}\ {\small\text{PM}}\qquad\mathrm{(C)}\ \text{3:30}\ {\small\text{PM}}\qquad\mathrm{(D)}\ \text{4:30}\ {\small\text{PM}}\qq What is the [[reciprocal]] of <math>\frac{1}{2}+\frac{2}{3}</math>?
    13 KB (2,025 words) - 13:56, 2 February 2021
  • ...qquad\mathrm{(B)}\ \text{3:00}\ {\small\text{PM}}\qquad\mathrm{(C)}\ \text{3:30}\ {\small\text{PM}}\qquad\mathrm{(D)}\ \text{4:30}\ {\small\text{PM}}\qq ...8:30}=\text{2:40}</math> hours. Thus, the entire job is completed in <math>3\cdot(\text{2:40})=\text{8:00}</math> hours.
    1 KB (197 words) - 17:18, 4 June 2021
  • What is the [[reciprocal]] of <math>\frac{1}{2}+\frac{2}{3}</math>? ...mathrm{(B)}\ \frac{7}{6}\qquad\mathrm{(C)}\ \frac{5}{3}\qquad\mathrm{(D)}\ 3\qquad\mathrm{(E)}\ \frac{7}{2}</math>
    717 bytes (93 words) - 12:22, 7 September 2021
  • what is the [[coefficient]] of <math>x^{28}</math>? ...<math>x^{28}</math>. Thus the coefficient of the <math>x^{28}</math> term is <math>224 \Rightarrow \boxed{C}</math>.
    5 KB (895 words) - 22:54, 9 January 2021
  • ...sitive integers <math>a_1 \le 2008</math> is it true that <math>a_1</math> is less than each of <math>a_2</math>, <math>a_3</math>, and <math>a_4</math>? ...4n+1</math>, <math>4n+2</math>, or <math>4n+3</math>, where <math>n</math> is a nonnegative integer.
    2 KB (386 words) - 13:52, 21 December 2020
  • ...<math>\frac{x}{2}</math>, and Carlos's lawn has an area of <math>\frac{x}{3}</math>. ...ed of <math>\frac{y}{3}</math>, and Beth's cuts at a speed <math>\frac{2y}{3}</math>.
    2 KB (380 words) - 15:00, 17 August 2023
  • What is the [[volume]] of a [[cube]] whose [[surface area]] is twice that of a cube with volume 1? A cube with volume <math>1</math> has a side of length <math>\sqrt[3]{1}=1</math> and thus a surface area of <math>6 \cdot 1^2=6</math>.
    1 KB (196 words) - 04:49, 16 January 2023
  • ...h>12^\text{th}</math> term of the sequence is <math>\log{b^n}</math>. What is <math>n</math>? ...</math>, and <math>8A + 15B</math>, and the <math>12^\text{th}</math> term is <math>nB</math>.
    3 KB (577 words) - 16:33, 9 October 2022
  • Which of the following is equal to the [[product]] Thus, the product is <math>\frac{2008}{4}=502</math>, and the answer is <math>\textbf{(B)}</math>.
    1 KB (145 words) - 04:52, 21 July 2022
  • .../math> is <u>heavy-tailed</u> if <math>a_1 + a_2 < a_4 + a_5</math>. What is the number of heavy-tailed permutations? ..._2,a_3,a_4,a_5)</math> such that <math>a_1 + a_2 < a_4 + a_5</math>, there is exactly one permutation such that <math>a_1 + a_2 > a_4 + a_5</math>. Thus
    3 KB (409 words) - 01:21, 31 July 2023
  • ...nt to both <math>\overline{OA}</math> and <math>\overline{OB}</math>. What is the ratio of the area of the smaller circle to that of the larger circle? ...A=(3,0), B=(3/2,3/2*3^.5), C=(3^.5,1), D=(3^.5,0), F=(1.5*3^.5,1.5), G=(2*3^.5,2);
    4 KB (630 words) - 20:32, 4 June 2021
  • ...math>, and the area of <math>\triangle AKD</math> is <math>24.</math> What is the area of trapezoid <math>ABCD</math>? pair D=(0,0),C=(12,0), K=(7,16/3); /* note that K.x is arbitrary, as generator for A,B */
    2 KB (294 words) - 21:53, 17 October 2023
  • ...e positive <math>z</math>-axis. Let <math>O</math> be the [[origin]]. What is the volume of [[tetrahedron]] <math>OABC</math>? draw((0,10)--(0,0)--(8,0));draw((-3,-4)--(0,0));draw((0,10)--(-3,-4)--(8,0)--cycle);
    2 KB (302 words) - 04:51, 16 January 2023
  • ...f{(C)}\ [0,2],[-1,0]\qquad\mathrm{(D)}\ [1,3],[-1,0]\qquad\mathrm{(E)}\ [1,3],[0,1]</math> ...(x)</math> is defined if <math>f(x + 1)</math> is defined. Thus the domain is all <math>x| x + 1 \in [0,2] \rightarrow x \in [ - 1,1]</math>.
    991 bytes (154 words) - 01:21, 4 February 2019
  • ...ant rate of 4 miles per hour while LeRoy bails water out of the boat. What is the slowest rate, in gallons per minute, at which LeRoy can bail if they ar We set up the following equation, where <math>x</math> is the answer:
    2 KB (313 words) - 18:08, 4 June 2021
  • ...math> have radii <math>r_a</math> and <math>r_b</math>, respectively. What is <math>r_a/r_b</math>? ...uad\mathrm{(D)}\ \frac{5}{56}\left(10-\sqrt{2}\right)\\\mathrm{(E)}\ \frac{3}{28}\left(10-\sqrt{2}\right)</math>
    6 KB (951 words) - 16:31, 2 August 2019
  • ...i=0</math> are the vertices of a convex polygon in the complex plane. What is the area of the polygon? ...hrm{(C)}\ 2\qquad\mathrm{(D)}\ 2^{\frac{5}{4}}\qquad\mathrm{(E)}\ 2^{\frac{3}{2}}</math>
    1 KB (199 words) - 01:38, 10 November 2019
  • Let <math>k={2008}^{2}+{2}^{2008}</math>. What is the units digit of <math>k^2+2^k</math>? ...th>k^2 \equiv 0 \pmod{10}</math>. Since <math>k = 2008^2+2^{2008}</math> is a multiple of four and the units digit of powers of two repeat in cycles of
    4 KB (547 words) - 04:19, 30 September 2023
  • ...that the inner corners each touch an inner corner of an adjacent mat. What is <math>x</math>? path mat=(-2.687,-1.5513)--(-2.687,1.5513)--(-3.687,1.5513)--(-3.687,-1.5513)--cycle;
    10 KB (1,518 words) - 17:00, 16 May 2023
  • ...t the <math>13</math> visible numbers have the greatest possible sum. What is that sum? ...ubes each have a pair of opposite faces that are covered up. When the cube is folded, <math>(1,32)</math>; <math>(2,16)</math>; and <math>(4,8)</math> ar
    2 KB (295 words) - 00:11, 19 April 2020
  • ...ath>\text{gcd}(a,b)=1</math> and <math>\frac{a}{b} + \frac{14b}{9a}</math> is an integer? ...| 9a^2 + 14b^2 \quad\Longrightarrow\quad 9 | b^2 \quad\Longrightarrow\quad 3 | b</math>
    9 KB (1,522 words) - 22:46, 12 May 2022
  • ...involving odd powers of <math>r</math> have a sum of <math>3</math>. What is <math>a+r</math>? ...}\ \frac{12}{7}\qquad\textbf{(C)}\ \frac{3}{2}\qquad\textbf{(D)}\ \frac{7}{3}\qquad\textbf{(E)}\ \frac{5}{2} </math>
    2 KB (250 words) - 15:41, 27 July 2021
  • What is the area of the region defined by the [[inequality]] <math>|3x-18|+|2y+7|\l <math>\mathrm{(A)}\ 3\qquad\mathrm{(B)}\ \frac{7}{2}\qquad\mathrm{(C)}\ 4\qquad\mathrm{(D)}\ \fra
    902 bytes (125 words) - 10:28, 13 March 2016
  • ...n on an older television screen with a <math>27</math>-inch diagonal. What is the height, in inches, of each darkened strip? ...B)}\ 2.25\qquad\mathrm{(C)}\ 2.5\qquad\mathrm{(D)}\ 2.7\qquad\mathrm{(E)}\ 3</math>
    2 KB (256 words) - 00:31, 19 October 2020
  • ...h>acb</math> lies two thirds of the way between the same two squares. What is <math>a+b+c</math>? The difference between <math>acb</math> and <math>abc</math> is given by
    5 KB (758 words) - 16:35, 15 February 2021
  • ...ven that <math>PQ=1</math>, <math>PR=2</math>, and <math>PS=3</math>, what is <math>AB</math>? ...\mathrm{(B)}\ 3\sqrt{3} \qquad \mathrm{(C)}\ 6 \qquad \mathrm{(D)}\ 4\sqrt{3} \qquad \mathrm{(E)}\ 9</math>
    3 KB (401 words) - 22:58, 8 May 2023
  • ...rrounded by <math>4</math> circles of radius <math>r</math> as shown. What is <math>r</math>? ...d\textbf{(B) } 1+\sqrt{2} \qquad\textbf{(C) } \sqrt{6} \qquad\textbf{(D) } 3 \qquad\textbf{(E) } 2+\sqrt{2}</math>
    2 KB (385 words) - 14:17, 4 June 2021
  • ...math>S</math> of <math>A</math> for which <math>\sigma(S) = n</math>. What is the smallest possible value of <math>a_{10}</math>? ...math> is a <math>2</math>-<math>3</math> set and <math>\{1,2,4,10\}</math> is a <math>4</math>-<math>8</math> set)
    5 KB (858 words) - 07:52, 19 July 2016
  • ...math>. Point <math>D</math> is the [[midpoint]] of <math>BC</math>. What is the largest possible value of <math>\tan{\angle BAD}</math>? ...}\ \frac{\sqrt{3}}{2\sqrt{2}}\qquad\mathrm{(D)}\ \frac{\sqrt{3}}{4\sqrt{2}-3}\qquad\mathrm{(E)}\ 1</math>
    3 KB (513 words) - 14:35, 7 June 2018
  • ...+ 1}) = (\sqrt {3}a_n - b_n, \sqrt {3}b_n + a_n)</math> for <math>n = 1,2,3,\ldots</math>. Suppose that <math>(a_{100},b_{100}) = (2,4)</math>. What is <math>a_1 + b_1</math>?
    5 KB (745 words) - 10:58, 9 December 2022
  • ...th> is between <math>\frac{1}{3}</math> and <math>\frac{2}{3}</math>. What is the area of <math>R</math>? \textbf{(D)}\ \frac{3-\sqrt3}{9} \qquad
    2 KB (328 words) - 10:54, 4 July 2013
  • ...watch the light. What is the probability that the color changes while she is watching? ...frac{1}{10} \qquad \mathrm{(D)}\ \frac{1}{7} \qquad \mathrm{(E)}\ \frac{1}{3}</math>
    1 KB (166 words) - 01:03, 19 October 2020
  • ...> by taping <math>\overline{AB}</math> to <math>\overline{DC}</math>. What is <math>\sin(\angle ABC)</math>? ...pi}{6} \qquad \mathrm{(D)}\ \frac{\pi}{4} \qquad \mathrm{(E)}\ \frac{\sqrt{3}}{2}</math>
    1 KB (166 words) - 16:35, 15 February 2021
  • ...f these base-<math>3</math> representations are palindromes? (A palindrome is a number that reads the same forward and backward.) <math>2007_{10} = 2202100_{3}</math>
    2 KB (267 words) - 16:35, 15 February 2021
  • ...tracts 1. What is the probability that the fourth term in Jacob's sequence is an [[integer]]? ...{(C)}\ \frac{1}{2}\qquad\mathrm{(D)}\ \frac{5}{8}\qquad\mathrm{(E)}\ \frac{3}{4}</math>
    2 KB (321 words) - 17:23, 9 November 2022
  • ...ositive integer leg lengths have areas that are numerically equal to <math>3</math> times their perimeters? We have <math>\frac{1}{2}ab = 3(a+b+c)</math>.
    4 KB (725 words) - 19:59, 4 January 2024
  • Each face of a regular tetrahedron is painted either red, white, or blue. Two colorings are considered indistingu <math>4:0:0</math>, <math>3:1:0</math>, <math>2:2:0</math>, or <math>2:1:1</math>
    3 KB (463 words) - 16:35, 15 February 2021
  • ...adius <math>1</math> and passes through the center of <math>D</math>. What is the radius of circle <math>B</math>? pair A=(-1,0),B=(2/3,8/9),C=(2/3,-8/9),D=(0,0);
    5 KB (785 words) - 00:29, 31 July 2023
  • ...us <math>1</math> in the plane that cover <math>\overline{AB}</math>. What is the area of <math>S</math>? ...textbf {(C) } 3\pi - \frac {\sqrt3}{2} \qquad \textbf {(D) } \frac {10\pi}{3} - \sqrt3 \qquad \textbf {(E) }4\pi - 2\sqrt3</math>
    3 KB (434 words) - 22:25, 22 November 2021
  • ...askets during a game. Each basket was worth either <math>2</math> or <math>3</math> points. How many different numbers could represent the total points <math>\mathrm{(A)}\ 2\qquad\mathrm{(B)}\ 3\qquad\mathrm{(C)}\ 4\qquad\mathrm{(D)}\ 5\qquad\mathrm{(E)}\ 6</math>
    12 KB (1,838 words) - 16:52, 7 October 2022
  • ...askets during a game. Each basket was worth either <math>2</math> or <math>3</math> points. How many different numbers could represent the total points <math>\textbf{(A)}\ 2 \qquad \textbf{(B)}\ 3 \qquad \textbf{(C)}\ 4 \qquad \textbf{(D)}\ 5 \qquad \textbf{(E)}\ 6</math>
    14 KB (2,199 words) - 13:43, 28 August 2020
  • A <math>4\times 4</math> block of calendar dates is shown. First, the order of the numbers in the second and the fourth rows ar 1&2&3&4\\\hline
    2 KB (219 words) - 23:53, 31 July 2023
  • ...\angle AOC = 30^\circ</math>, and <math>\angle DOB = 45^\circ</math>. What is the ratio of the area of the smaller sector <math>COD</math> to the area of pair C = 3*dir (30);
    1 KB (183 words) - 22:35, 10 June 2017
  • ...ural number with exactly 6 positive divisors, the sum of whose reciprocals is 2. Find that natural number. ...p_1</math> and <math>p_2</math> be primes. Therefore, one of the following is true:
    2 KB (279 words) - 19:14, 10 March 2015
  • <math>x^2-2y^2=1</math> means that <math>x</math> is odd. We can let <math>x=2x_1-1</math> for some <math>x_1>0</math>: y is even, <math>y=2y_1</math> for some <math>y_1>0</math>.
    3 KB (478 words) - 23:41, 5 January 2014
  • ...urnament]], one of two tests given to choose the [[Alabama ARML]] team. It is similar to an easy [[AIME]], Alabaman scores average about 5 right, while t * [[2006 Alabama ARML TST Problems/Problem 3]]
    1 KB (167 words) - 01:00, 19 June 2018
  • ==Problem 3== River draws four cards from a standard 52 card deck of playing cards. Exactly 3 of them are 2’s. Find the probability River drew exactly one spade and on
    5 KB (769 words) - 20:56, 24 March 2015
  • ...rs to buy flowers for a classmate who is in the hospital. Roses cost <math>3</math> dollars each, and carnations cost <math>2</math> dollars each. No ot
    885 bytes (131 words) - 13:47, 15 February 2021
  • ...math>\frac{1}{8}</math> of the volume of the mountain is above water. What is the depth of the ocean at the base of the mountain in feet? ...of the cone formed by cutting off the mountain at <math>4,000</math> feet is half that of the original mountain). Therefore, volume varies as the invers
    2 KB (251 words) - 22:12, 8 May 2021
  • ...askets during a game. Each basket was worth either <math>2</math> or <math>3</math> points. How many different numbers could represent the total points <math>\textbf{(A)}\ 2 \qquad \textbf{(B)}\ 3 \qquad \textbf{(C)}\ 4 \qquad \textbf{(D)}\ 5 \qquad \textbf{(E)}\ 6</math>
    1 KB (176 words) - 16:02, 19 August 2017
  • ...om the line segment joining <math>(0,1)</math> to <math>(2,1)</math>. What is the probability that circles <math>A</math> and <math>B</math> intersect? ...)} \; \frac {2 + \sqrt {3}}{4} \qquad \textbf{(E)} \; \frac {4 \sqrt {3} - 3}{4}</math>
    6 KB (1,008 words) - 11:46, 24 December 2020
  • ...oint <math>C</math> is the midpoint of the minor arc <math>AB</math>. What is the length of the line segment <math>AC</math>? ...ot 5\cdot \frac{4}{5}} = \sqrt{50-50\frac{4}{5}} = \sqrt{10}</math>, which is answer choice <math>\boxed{\text{A}}</math>.
    2 KB (319 words) - 13:48, 15 February 2021
  • ...sing order. The number of ways to take three digits out is <math>\binom{9}{3}=\boxed{84}</math> ...ay that <math>b</math> is the second smallest integer. Then <math>b</math> is the ten-thousands digit. etc.
    1 KB (200 words) - 19:37, 12 April 2012
  • ...ath>\angle B</math> and <math>\angle C</math> meet at <math>Q</math>. What is the area of hexagon <math>ABQCDP</math>? ...C)}\ 32\sqrt{3}\qquad \textbf{(D)}\ 35\sqrt{3}\qquad \textbf{(E)}\ 36\sqrt{3}</math>
    12 KB (2,015 words) - 20:54, 9 October 2022
  • ...ve integer <math>n,\ A_{n-1}B_nA_n</math> is an equilateral triangle. What is the least <math>n</math> for which the length <math>A_0A_n\geq100</math>? ...lateral triangle, can be compacted as follows: <cmath>\left(a_n\frac{\sqrt{3}}{2}\right)^2=\frac{a_n}{2}+a_{n-1}+a_{n-2}+\cdots+a_1</cmath>
    9 KB (1,482 words) - 13:52, 4 April 2024
  • ...er SUV, which requires 2 adjacent spaces. What is the probability that she is able to park? ...4}{7} \qquad \textbf{(C)} \; \frac {81}{140} \qquad \textbf{(D)} \; \frac {3}{5} \qquad \textbf{(E)} \; \frac {17}{28}</math>
    4 KB (653 words) - 11:06, 15 October 2022
  • <math>\frac{3\times 5}{9\times 11}\times \frac{7\times 9\times 11}{3\times 5\times 7}=</math> == Problem 3 ==
    12 KB (1,670 words) - 17:42, 24 November 2021
  • ...m^3 kg^{-1} s^{-2}</math>. It is commonly denoted as <math>G</math>. There is some uncertainty associated with the value of the constant, due to limitati ...large mountain", thus estimating <math>G = (6.7 \pm 0.6) \cdot 10^{-11} m^3 kg^{-1} s^{-2}</math>.
    2 KB (274 words) - 12:56, 9 June 2021
  • ...004}</math> is divided by <math>1000</math>, a remainder of <math>N</math> is obtained. Determine the value of <math>N</math>. ...= 400</math>, by [[Fermat's Little Theorem|Fermat-Euler's Theorem]], this is equivalent to finding <math>\frac{7^{400 \cdot 5 + 5} - 1}{6} \equiv \frac{
    685 bytes (81 words) - 10:51, 11 June 2013
  • # None of the first four letters is an <math>A</math>. # None of the next five letters is a <math>B</math>.
    1 KB (221 words) - 17:27, 23 February 2013
  • ...</math> denote the apex of <math>P</math>. If the volume of <math>P</math> is eight times that of <math>P'</math>, then the value of <math>XT</math> can ...with volumes, the ratio of the volume of <math>P'</math> to <math>P</math> is the cube of the ratio of the height of <math>P'</math> to <math>P</math>.
    3 KB (446 words) - 00:18, 10 February 2020
  • is larger. It is [[Trivial Inequality|trivial]] that
    2 KB (393 words) - 07:54, 19 July 2016
  • ...by <math>20</math> more boy students, all of whom like to dance, the party is now <math>58\%</math> girls. How many students now at the party like to dan ...the area common to triangle <math>GEM</math> and square <math>AIME</math> is <math>80</math> square units. Find the length of the altitude to <math>EM<
    9 KB (1,536 words) - 00:46, 26 August 2023
  • ...nd subtractions alternate in pairs. Find the remainder when <math>N</math> is divided by <math>1000</math>. ...te break at the end of every mile. Jennifer bikes at a constant rate which is three-quarters the rate that Rudolph bikes, but Jennifer takes a five-minut
    7 KB (1,167 words) - 21:33, 12 August 2020
  • ...by <math>20</math> more boy students, all of whom like to dance, the party is now <math>58\%</math> girls. How many students now at the party like to dan ...gives <math>k = 116</math>. Thus, the number of people that like to dance is <math>2k + 20 = \boxed{252}</math>.
    2 KB (378 words) - 22:23, 19 December 2022
  • ...the area common to triangle <math>GEM</math> and square <math>AIME</math> is <math>80</math> square units. Find the length of the altitude to <math>EM< ..., then the maximum area of the intersection of the triangle and the square is <math>5\cdot10=50</math>.
    2 KB (250 words) - 19:37, 29 December 2020
  • ...ath> kilometers after jogging for <math>2</math> hours, swimming for <math>3</math> hours, and biking for <math>4</math> hours. Their biking, jogging, ..., we need <math>s\equiv1\pmod{4}</math>. Thus, <math>(j,s) = (13,1),(8,5),(3,9)</math>.
    3 KB (564 words) - 22:15, 28 November 2023
  • ..., one of the factors is even. A [[parity]] check shows that if one of them is even, then both must be even. Since <math>244 = 2^2 \cdot 61</math>, the fa Indeed, by solving, we find <math>(x,y) = (18,62)</math> is the unique solution.
    4 KB (732 words) - 22:17, 28 November 2023
  • ...]] array of numbers has a first row consisting of the odd integers <math>1,3,5,\ldots,99</math> in increasing order. Each row below the first has one f ...ontains one entry that is a multiple of <math>67</math>, and so the answer is <math>\frac{33+1}{2} = \boxed{017}</math>.
    3 KB (509 words) - 17:21, 22 March 2018
  • <cmath>\arctan\frac {1}{3} + \arctan\frac {1}{4} + \arctan\frac {1}{5} + \arctan\frac {1}{n} = \frac Applying this to the first two terms, we get <math>\arctan{\dfrac{1}{3}} + \arctan{\dfrac{1}{4}} = \arctan{\dfrac{7}{11}}</math>.
    3 KB (490 words) - 22:36, 28 November 2023
  • ...Let <math>\frac {m}{n}</math> be the probability that the stack of crates is exactly <math>41\mathrm{ft}</math> tall, where <math>m</math> and <math>n</ Only the heights matter, and each crate is either 3, 4, or 6 feet tall with equal probability. We have the following:
    6 KB (909 words) - 15:39, 8 August 2022
  • ...<math>m</math> and <math>n</math> are positive integers and <math>n</math> is not divisible by the square of any prime. Find <math>m + n</math>. ...A=(0,0), D=(4,0), B= A+2 expi(1/3*pi), C= D+2expi(2/3*pi), E=(-4/3,0), F=(3,0);
    4 KB (629 words) - 22:38, 28 November 2023
  • ...AA</math>, <math>B</math>, and <math>AABAA</math>, while <math>BBAB</math> is not such a sequence. How many such sequences have length 14? 3&1&2&
    7 KB (1,173 words) - 22:39, 28 November 2023
  • ...s the photoelectric eye in one hour. Find the quotient when <math>M</math> is divided by <math>10</math>. ...unit, the CAR comes first, THEN the empty space. So at time zero, the car is right at the eye.
    4 KB (669 words) - 18:35, 8 October 2023
  • ...a_1x + a_2y + a_3x^2 + a_4xy + a_5y^2 + a_6x^3 + a_7x^2y + a_8xy^2 + a_9y^3.</cmath> There is a point <math>\left(\frac {a}{c},\frac {b}{c}\right)</math> for which <math
    8 KB (1,218 words) - 00:07, 11 April 2024
  • ...th>n</math> are positive integers, <math>m<1000</math>, and <math>m</math> is not divisible by the <math>n</math>th power of any prime. Find <math>m+n</m ...er, and <math>M</math> and <math>N</math> be the two points whose distance is <math>\sqrt{17}</math> from <math>P</math>. Also, let <math>R</math> be the
    6 KB (1,041 words) - 00:54, 1 February 2024
  • New in 2016 is the option of a Computer Science Track July 12-22 only for students enterin ...by Dr. Kevin Hopkins, the chairman of the math department at SBU, but SBU is not a sponsor of the camp.
    2 KB (343 words) - 22:29, 23 January 2016
  • ...ne <math>CT</math> is tangent to <math>\omega</math>. Point <math>P</math> is the foot of the perpendicular from <math>A</math> to line <math>CT</math>. ...math>k = \frac{2x-27}{x^2} \Longrightarrow kx^2 - 2x + 27 = 0</math>; this is a quadratic, and its [[discriminant]] must be nonnegative: <math>(-2)^2 - 4
    8 KB (1,333 words) - 00:18, 1 February 2024
  • Function <math>g(y)</math> is given such way that for all <math>y</math>, ...{abcde}</math> and last two digits on the <math>\text {fghij}</math>? That is, <math>b + c + d + i + j</math>.
    6 KB (909 words) - 07:27, 12 October 2022
  • ...60401 + 256 = 104060657, \end{aligned}</math> and so the sum of the digits is <math>1+4+6+6+5+7 = \boxed{29}.</math>
    517 bytes (55 words) - 20:01, 23 March 2017
  • If <math>x</math> is an odd number, then find the largest integer that always divides the expres <cmath>4(5x + 1)(5x + 3)(5x+5)</cmath>
    1 KB (179 words) - 00:04, 10 August 2019
  • : <math>\frac n2</math> is a perfect square. : <math>\frac n3</math> is a perfect cube.
    2 KB (323 words) - 00:32, 31 August 2015
  • What is the coefficient of <math>x^3y^{13}</math> in <math>\left(\frac 12x + y\righ ...^n</math> has 3 consecutive terms with coefficients in the ratio <math>1:2:3</math> that can be written in the form <cmath>{n\choose k} : {n\choose k+1}
    6 KB (992 words) - 14:15, 13 February 2018
  • ...nd subtractions alternate in pairs. Find the remainder when <math>N</math> is divided by <math>1000</math>. ...- 93^2 + 92^2 + 91^2 + \ldots - 10^2 - 9^2 + 8^2 + 7^2 - 6^2 - 5^2 + 4^2 + 3^2 - 2^2 - 1^2 \mbox{, and reordering, we get}\\
    4 KB (575 words) - 16:41, 14 April 2024
  • ...te break at the end of every mile. Jennifer bikes at a constant rate which is three-quarters the rate that Rudolph bikes, but Jennifer takes a five-minut ...>r</math> be the time Rudolph takes disregarding breaks and <math>\frac{4}{3}r</math> be the time Jennifer takes disregarding breaks. We have the equati
    3 KB (432 words) - 11:53, 19 June 2019
  • ...se. The individual slices are not necessarily parallel to each other. What is the maximum possible volume in cubic cm of the remaining block of cheese af ...\boxed{729}</math>. Equality is achieved when <math>a=b=c=9</math>, which is possible if we make one slice perpendicular to the <math>10</math> cm edge,
    2 KB (282 words) - 12:56, 3 March 2015
  • ...th>3</math>, we find that <math>\overline{2008}_{10} = \overline{2202101}_{3}</math>. In other words, ...2008 = 2 \cdot 3^{6} + 2 \cdot 3^{5} + 2 \cdot 3^3 + 1 \cdot 3^2 + 1 \cdot 3^0</math></center>
    1 KB (167 words) - 21:47, 21 September 2020
  • ...th>, note that the midpoint of <math>\overline{AD}</math>, <math>N</math>, is the center of the [[circumcircle]] of <math>\triangle AED</math>. We can do ...ath>, <math>M</math>, to the midpoint of <math>\overline{AD}</math>, which is <math>N</math>, then <math>E,M,N</math> are [[collinear]].
    8 KB (1,338 words) - 23:15, 28 November 2023
  • The sequence <math>\{a_n\}</math> is defined by The sequence <math>\{b_n\}</math> is defined by
    1 KB (223 words) - 23:34, 4 July 2013
  • 8x^3 + 1001x + 2008 = 0. Find <math>(r + s)^3 + (s + t)^3 + (t + r)^3</math>.
    7 KB (1,251 words) - 19:18, 2 January 2024
  • is an integer. ...\cdot 251</math>. Therefore, it is clear that <math>n = \boxed{251}</math> is the smallest such integer.
    4 KB (621 words) - 18:26, 16 January 2023
  • ...direction. Given that the particle's position after <math>150</math> moves is <math>(p,q)</math>, find the greatest integer less than or equal to <math>| ...ta</math> be the inclination of OP to the x-axis. If <math>(x', y')</math> is the position of the particle after a move from <math>P</math>, then we have
    5 KB (725 words) - 22:37, 28 January 2024
  • ...w shows a <math>4\times4</math> rectangular array of points, each of which is <math>1</math> unit away from its nearest neighbors. ...the property that the distance between consecutive points of the sequence is strictly increasing. Let <math>m</math> be the maximum possible number of p
    4 KB (569 words) - 09:44, 25 November 2019
  • ...math>n</math>, and <math>k</math> are positive integers and <math>k</math> is the product of distinct primes. Find <math>m + nk</math>. pair A=(0,96),B=(-28,0),C=(28,0),X=C-(64/3,0),Y=B+(4*r/3,0),P=X+(0,16),Q=Y+(0,r),M=foot(Q,X,P);
    6 KB (1,065 words) - 20:12, 9 August 2022
  • ...flags on either pole are adjacent. Find the remainder when <math>N</math> is divided by <math>1000</math>. ...ns where either pole has no flags, we have to count these separately. This is the same as choosing our extra blue as one of the two ends, and ordering th
    10 KB (1,550 words) - 12:58, 15 July 2023
  • :(ii) <math>2n + 79</math> is a perfect square. ...h>, or equivalently, <math>(2n + 1)(2n - 1) = 4n^2 - 1 = 12m^2 + 12m + 3 = 3(2m + 1)^2</math>.
    4 KB (628 words) - 16:23, 2 January 2024
  • An '''ultrafilter''' is a [[set theory | set theoretic]] structure. An ultrafilter on a set <math>X</math> is a non-empty [[filter]] <math>\mathcal{F}</math> on <math>X</math> with the
    9 KB (1,685 words) - 20:28, 13 October 2019
  • pair P=(7,0), Q=(14,0), R=P+7expi(pi/3), M=(10,1.2); We rotate figure <math>PRQM</math> by a clockwise angle of <math>\pi/3</math> about <math>Q</math> to obtain figure <math>RR'QM'</math>:
    7 KB (1,221 words) - 18:57, 3 July 2013
  • ...ath>B_1</math> in such a way that convex quadrilateral <math>APCB_1</math> is cyclic, <math>QB_1 \parallel BA </math>, and <math>B_1 </math> and <math>Q ...f the circumcircle of <math>B_1' C_1P</math> and line <math>BC</math>. It is enough to show that <math>B_1'=B_1</math> and <math>Q' =Q</math>. All our
    6 KB (1,117 words) - 01:17, 11 October 2021
  • \begin{align*}x^6 + x^3 + x^3y + y &= 147^{157} \\ x^3 + x^3y + y^2 + y + z^9 &= 157^{147}\end{align*}
    7 KB (1,053 words) - 10:38, 12 August 2015
  • The '''Asian Pacific Mathematics Olympiad''' ('''APMO''') is a mathematics olympiad for high school students in countries on and near th ...|type=Proof|difficulty=6 - 8|breakdown=<u>Problem 1</u>: 6<br><u>Problem 2/3</u>: 7<br><u>Problem 4</u>: 7.5<br><u>Problem 5</u>: 8}}
    1 KB (196 words) - 10:24, 12 October 2020
  • ...th>(1+x_1)(1+x_2)(1+x_3)\cdots (1+x_n)\leq 1+S+\dfrac{S^2}{2!}+\dfrac{S^3}{3!}+\cdots +\dfrac{S^n}{n!}</cmath>. == Problem 3 ==
    2 KB (340 words) - 10:27, 11 April 2016
  • ...g'') Determine all composite positive integers <math>n</math> for which it is possible to arrange all divisors of <math>n</math> that are greater than 1 ...</math>, start with the circular arrangement of <math>n,p_{1}p_{2},p_{2}p_{3}\ldots,p_{k-1}p_{k}</math> as shown.
    4 KB (650 words) - 13:40, 4 July 2013
  • ...math>, how many non-similar triangles are there in which <math>AEGF</math> is a cyclic quadrilateral? == Problem 3 ==
    2 KB (364 words) - 23:07, 12 September 2016
  • ...and fix a cuboid, <b>keeping the surfaces of the cuboid red</b>. Now what is the maximum possible volume of the cuboid? ...nt 1:''' The number of cubes with at least one red face is <math>5^3-(5-2)^3=125-27=98</math>. Therefore, the volume of the cube cannot more than <math>
    2 KB (283 words) - 21:47, 13 July 2023
  • ...or a [[circle]] of [[radius]] <math>r</math>, the curvature at every point is <math>\frac{1}{r}</math>. Intuitively, this grows smaller as <math>r</math ...<math>y = f(x)</math> of the function at the point <math>(x, f(x))</math> is given by the formula
    1 KB (196 words) - 21:53, 20 November 2015
  • ...ea of <math>\triangle DEF</math> to the area of <math>\triangle ABC</math> is ...1}{6} } \qquad \mathrm{(B) \ \frac {1}{4} } \qquad \mathrm{(C) \ \frac {1}{3} } \qquad \mathrm{(D) \ \frac {2}{5} } \qquad \mathrm{(E) \ \frac {1}{2} }
    1 KB (200 words) - 20:58, 10 February 2019
  • .... Thus, the cone is the special case of the [[pyramid]] in which the base is circular. import three; currentprojection = perspective(0,-3,1); defaultpen(linewidth(0.7)); triple vertex = (0,0,1.5);
    7 KB (1,128 words) - 20:12, 27 September 2022
  • <math>\sqrt[4]{x+27}=1\Rightarrow x=-26, \sqrt[4]{55-x}=\sqrt[4]{81}=3</math> <math>\sqrt[4]{x+27}=3\Rightarrow x=54, \sqrt[4]{55-x}=1</math>
    1 KB (167 words) - 09:56, 15 December 2018
  • ...urnament]], one of two tests given to choose the [[Alabama ARML]] team. It is similar to an easy [[AIME]], Alabaman scores average about 5 right, while t * [[2007 Alabama ARML TST Problems/Problem 3]]
    1 KB (167 words) - 00:59, 19 June 2018
  • ...a^2</math>, so <math>b^2 \ge \frac {3}{4}a^2 \implies \rho^2 \le \frac {4}{3}</math>. ...e equation, so <math>\rho^2 = \frac {4}{3}</math>, and the answer is <math>3 + 4 = \boxed{007}</math>.
    8 KB (1,387 words) - 11:56, 29 January 2024
  • ...plex plane]] has opposite pairs of sides one unit apart. One pair of sides is parallel to the imaginary axis. Let <math>R</math> be the region outside th ...{cis}\, \theta = \cos \theta + i \sin \theta</math>). Since <math>R</math> is symmetric every <math>60^{\circ}</math> about the origin, it suffices to co
    6 KB (894 words) - 18:56, 25 December 2022
  • ...greatly simplify the proofs of many theorems concerning [[polygon]]s, and is helpful in solving complex geometry problems involving lengths. In essence ...h>AB</math> with point <math>C</math> on it, and the mass put on a point P is denoted as <math>m_P</math>,
    5 KB (815 words) - 22:31, 1 April 2024
  • The '''Shoelace Theorem''' is a nifty formula for finding the [[area]] of a simple [[polygon]] given the ...sted in clockwise order. Then the area (<math>A</math>) of <math>P</math> is
    8 KB (1,358 words) - 15:32, 22 February 2024
  • <math>\triangle DEF</math> is inscribed inside <math>\triangle ABC</math> such that <math>D,E,F</math> li draw(A--E,StickIntervalMarker(1,3,size=6));draw(B--D,StickIntervalMarker(1,3,size=6));
    2 KB (265 words) - 20:02, 29 January 2020
  • <math>0.999\ldots</math> (or <math>0.\overline{9}</math>) is an equivalent representation of the [[integer]] <math>1</math>. ...roblem, one needs mathematics beyond the elementary school level math that is needed to understand the question.
    3 KB (577 words) - 20:04, 4 February 2023
  • ...ath> is a [[positive]] [[real number]]. Which is equivalent to <math>\sqrt[3]{x\sqrt{x}}</math>? <math>\mathrm{(A)}\ x^{1/6}\qquad\mathrm{(B)}\ x^{1/4}\qquad\mathrm{(C)}\ x^{3/8}\qquad\mathrm{(D)}\ x^{1/2}\qquad\mathrm{(E)}\ x</math>
    707 bytes (97 words) - 19:54, 24 April 2024
  • An equilateral triangle of side length <math>10</math> is completely filled in by non-overlapping equilateral triangles of side lengt ...s <math>\frac{10^2\sqrt3}{4}</math>, while the area of each small triangle is <math>\frac{1^2\sqrt3}{4}</math>. Dividing these two quantities results in
    2 KB (323 words) - 20:23, 14 July 2021
  • ...dollars to buy flowers for a classmate who is in the hospital. Roses cost 3 dollars each, and carnations cost 2 dollars each. No other flowers are to b ...Of course, the number of roses must be such that the number of carnations is non-negative. We get the inequality <math>25-3r \geq 0</math> which solves
    2 KB (401 words) - 20:32, 26 July 2022
  • ...atic equation <math>ax^2 - 2ax + b = 0</math> has two real solutions. What is the average of these two solutions? ...formulas, the sum of the roots is <math>2</math>, therefore their average is <math>1\Rightarrow \boxed{A}</math>.
    1 KB (218 words) - 16:59, 13 April 2024
  • ...<math>C</math> is the [[midpoint]] of the minor arc <math>AB</math>. What is the length of the line segment <math>AC</math>? ...h>\overline{AB}</math> and <math>\overline{OC}</math> (then <math>D</math> is the midpoint of <math>\overline{AB}</math>). <math>OA=OB=5</math>, since th
    1 KB (247 words) - 12:36, 7 June 2021
  • Suppose that <math>(u_n)</math> is a [[sequence]] of real numbers satifying <math>u_{n+2}=2u_{n+1}+u_n</math>, and that <math>u_3=9</math> and <math>u_6=128</math>. What is <math>u_5</math>?
    913 bytes (135 words) - 19:42, 24 October 2022
  • ...an of the first <math>n</math> terms of a sequence is <math>n</math>. What is the <math>2008^{\text{th}}</math> term of the sequence? ...08^2</math>. Hence, the <math>2008^{\text{th}}</math> term of the sequence is <math>2008^2-2007^2=(2008+2007)(2008-2007)=4015\Rightarrow \boxed{\textbf{(
    2 KB (387 words) - 05:10, 1 October 2023
  • ...</math> and <math>\angle AOB=30^\circ</math>. Suppose that <math>OA</math> is rotated <math>90^\circ</math> counterclockwise about <math>O</math>. What a \mathrm{(A)}\ \left( - \frac {10}{3}\sqrt {3},5\right)
    3 KB (444 words) - 04:45, 21 August 2023
  • ...ht side of the equation is odd. <math>2b</math> is even. <math>2b+1</math> is odd. So <math>a=1,3,5,7,9,11,13</math>, but if <math>a=1</math>, then <math>b=0</math>. Thus <m
    1,016 bytes (166 words) - 11:39, 8 October 2023
  • ...t the sum of the die rolls is odd? (Note that if no die is rolled, the sum is 0.) ...\qquad \mathrm{(D)}\ {{{\frac{5} {8}}}} \qquad \mathrm{(E)}\ {{{\frac{2} {3}}}}</math>
    2 KB (357 words) - 22:58, 4 October 2023
  • ...rk. On three separate occasions a pollster selects a voter at random. What is the probability that on exactly one of these three occasions the voter appr ...ach of these is <math>(0.7)(0.3)^2=0.063</math>. Thus, the answer is <math>3\cdot0.063=\boxed{\mathrm{(B)}\ {{{0.189}}}}</math>
    1 KB (202 words) - 12:42, 30 January 2024
  • ...ide. The tank is filled with water to a depth of <math>2</math> feet. What is the volume of water, in cubic feet? \mathrm{(B)}\ 24\pi - 24 \sqrt {3}
    2 KB (373 words) - 13:21, 7 June 2021
  • ..., <math>4</math>, <math>5</math>, <math>6</math>, and <math>8</math>. What is the probability that the sum of the top two numbers will be <math>5</math>, ...\ 5/18\qquad\mathrm{(B)}\ 7/18\qquad\mathrm{(C)}\ 11/18\qquad\mathrm{(D)}\ 3/4\qquad\mathrm{(E)}\ 8/9</math>
    4 KB (572 words) - 17:44, 14 June 2023
  • ...ouples are to sit in the chairs with men and women alternating, and no one is to sit either next to or across from his/her spouse. How many seating arran ...ssible arrangements for the five women. The answer is <math>10\cdot 4\cdot 3\cdot 2\cdot 1\cdot 2 = \boxed{(\text{C}) 480}</math>.
    2 KB (262 words) - 22:37, 6 November 2021
  • ...wo white beads, and one blue bead are placed in line in random order. What is the probability that no two neighboring beads are the same color? ...\ 1/12\qquad\mathrm{(B)}\ 1/10\qquad\mathrm{(C)}\ 1/6\qquad\mathrm{(D)}\ 1/3\qquad\mathrm{(E)}\ 1/2</math>
    4 KB (663 words) - 13:49, 7 June 2021
  • <math>\textbf{(A)}\ 1\qquad\textbf{(B)}\ 2\qquad\textbf{(C)}\ 3\qquad\textbf{(D)}\ 4\qquad\textbf{(E)}\ 5</math> ...the painted rectangle covers half the area as well. Since the border width is 1 foot, the dimensions of the rectangle are <math>a-2</math> by <math>b-2</
    2 KB (247 words) - 23:47, 17 October 2023
  • ...angle ABC = 70^\circ</math> and <math>m\angle BCD = 170^\circ</math>. What is the degree measure of <math>\angle BAD</math>? ...eir intersection point <math>Y</math>. Note that triangle <math>BCD</math> is an isosceles triangle so angles <math>CDB</math> and <math>CBD</math> are e
    12 KB (1,944 words) - 17:15, 20 January 2024
  • <cmath> \frac{a^2}{2ab^2-b^3+1} </cmath> is a positive integer.
    2 KB (397 words) - 00:23, 16 December 2017
  • ...gth to its width is <math>2:1</math>. What percent of the rectangle's area is inside the square? ...t <math>1</math>. So the percent of the rectangle's area inside the square is <math>\frac{1}{8} \times 100 = 12.5 \longrightarrow \fbox{A}</math>
    954 bytes (160 words) - 17:20, 4 June 2021
  • ...angle=1+2=3</math> and <math>\langle 12 \rangle =1+2+3+4+6=16</math>. What is <math>\langle\langle\langle 6\rangle\rangle\rangle</math>? Since <math>6</math> is a perfect number, any such operation where <math>n=6</math> will yield <mat
    969 bytes (143 words) - 15:12, 1 July 2021
  • ...math>S_2</math> to construct an even smaller square <math>S_3</math>. What is the area of <math>S_3</math>? ...\ \frac{1}{2}\qquad\mathrm{(B)}\ 1\qquad\mathrm{(C)}\ 2\qquad\mathrm{(D)}\ 3\qquad\mathrm{(E)}\ 4</math>
    1 KB (203 words) - 15:15, 1 July 2021
  • ...hour, and runs at a rate of 10 kilometers per hour. Which of the following is closest to the triathlete's average speed, in kilometers per hour, for the <math>\mathrm{(A)}\ 3\qquad\mathrm{(B)}\ 4\qquad\mathrm{(C)}\ 5\qquad\mathrm{(D)}\ 6\qquad\mathrm
    1 KB (209 words) - 22:01, 2 September 2020
  • ...th>I</math>, and set Han's speed as <math>H</math>. Therefore, Jan's speed is <math>H+5.</math> We get the following equation for how much Han is ahead of Ian:
    6 KB (1,033 words) - 15:19, 1 July 2021
  • ...ontaining all points that are outside the triangle but not more than <math>3</math> units from a point of the triangle? ...uad\mathrm{(D)}\ \left(2\sqrt{3}+3\right)^2\pi\\\mathrm{(E)}\ 9\left(\sqrt{3}+1\right)^2\pi</math>
    2 KB (348 words) - 19:59, 4 June 2021
  • ...riangle]] has [[perimeter]] <math>32</math> and area <math>20</math>. What is the length of its [[hypotenuse]]? ...he hypotenuse is <math>\sqrt{a^2+b^2}</math>, and the area of the triangle is <math>\frac 12 ab</math>. So we have the two equations
    8 KB (1,339 words) - 14:15, 1 August 2022
  • ...ise about the point <math>S</math> moved to after the first rotation. What is the length of the path traveled by point <math>P</math>? ...ad\mathrm{(C)}\ \left(3+\sqrt{10}\right)\pi\qquad\mathrm{(D)}\ \left(\sqrt{3}+2\sqrt{5}\right)\pi\\\mathrm{(E)}\ 2\sqrt{10}\pi</math>
    2 KB (321 words) - 17:52, 7 November 2021
  • ...site edges not containing <math>A</math> or <math>C</math>, as shown. What is the area of [[quadrilateral]] <math>ABCD</math>? ...hrm{(C)}\ \sqrt{2}\qquad\mathrm{(D)}\ \frac{5}{8}\qquad\mathrm{(E)}\ \frac{3}{4}</math>
    2 KB (300 words) - 19:42, 4 June 2021
  • ...ath>S=\lbrace a,b,c,d,e\rbrace</math> are to be chosen so that their union is <math>S</math> and their intersection contains exactly two elements. In how ...th of those subsets contain the two chosen elements, so their intersection is the two chosen elements). Unfortunately, we have over-counted (Take for ex
    4 KB (603 words) - 14:18, 10 November 2022
  • pair A=(1,2.7), B=(0,0), C=(3,0); pair S=IntersectionPoint(N--(N+3(I-N)),O,1);
    3 KB (437 words) - 15:47, 27 April 2008
  • ...all strictly greater than 1, such that <math>k_0 k_1 \dotsm k_n -1</math> is the product of two consecutive integers. for some integer <math>a</math>. Let <math>k_n = a^2 + 3a+3</math>. Evidently, <math>k_n >1</math>. Also,
    11 KB (1,964 words) - 03:38, 17 August 2019
  • Without Loss of Generality, assume <math>AB >AC</math>. It is sufficient to prove that <math>\angle OFA = 90^{\circ}</math>, as this woul Here, we used that <math>BM=MC</math>, as <math>M</math> is the midpoint of <math>BC</math>. Now, since <math>EC =EA</math> and <math>B
    20 KB (3,565 words) - 11:54, 1 May 2024
  • ...h>n - 2</math> triangles. If <math>\mathcal{P}</math> is regular and there is a triangulation of <math>\mathcal{P}</math> consisting of only isosceles tr ...angle in <math>Q</math>, from which it follows that the isosceles triangle is <math>\triangle P_aP_{a+k/2}\,P_{a+k}</math>, and so <math>2|k</math>. Repe
    8 KB (1,318 words) - 12:37, 20 April 2022
  • ...>n</math> paths (a partition of <math>S_n</math> into <math>m</math> paths is a set <math>\mathcal{P}</math> of <math>m</math> nonempty paths such that e ...-1,1)--(0,1)--(1,1)--(1,0),linewidth(0.7)); draw((1,0)--(1,-1),linetype("3 3")+linewidth(0.7)); draw((1,-1)--(1,-2)--(2,-2),linewidth(0.7));
    9 KB (1,585 words) - 01:00, 14 August 2014
  • ...mathematicians may be split between the two rooms is a power of two (i.e., is of the form <math>2^k</math> for some positive integer <math>k</math>). .../math> be an [[indicator function]] with <math>f(v) = 0</math> if a vertex is in the first [[partition]] and <math>f(v) = 1</math> otherwise (this corres
    13 KB (2,414 words) - 14:37, 11 July 2016
  • ...h>, all strictly greater than 1, such that <math>k_0k_1\cdots k_n-1</math> is the product of two consecutive integers. ===Problem 3===
    4 KB (674 words) - 21:48, 12 August 2014
  • ...ively. The [[quotient group]] of <math>{\rm G}</math> under this relation is often denoted <math>{\rm G/H}</math> (said, "<math>{\rm G}</math> mod <math An equivalent definition of normal subgroups is this-
    15 KB (2,840 words) - 12:22, 9 April 2019
  • A '''free magma''' is [[magma]] structure that is as general as possible&mdash;a magma generated from an initial set with no The free magma generated from a [[set]] <math>X</math> is constructed as follows.
    4 KB (887 words) - 13:19, 6 July 2016
  • ...2<br><u>MS Harder</u>: 2-3<br><u>HS Easier</u>: 0.5-3<br><u>HS Harder</u>: 3-5}} ...imple arithmetic problems and/or word problems. Every ten questions, there is an estimation question which requires the estimation answer to be within <m
    5 KB (801 words) - 12:47, 23 September 2023
  • ...e [[symmetric group]]; and for <math>n=3</math> or <math>n\ge 5</math>, it is in fact a [[simple group]]. <math>A_n</math> is also the group of [[determinant]]-preserving permutations of the rows of an
    776 bytes (127 words) - 11:47, 4 March 2022
  • A '''Mock ARML''' is a test written by an AoPS member, designed to mimic the format of the indiv Starting from 2015, trumpeter is releasing a Mock ARML every year some time from March to May of the year (s
    1 KB (153 words) - 11:44, 16 June 2019
  • ...range does not include negative numbers, it follows that the negative root is extraneous, and thus we have <math>x = \boxed{\frac{1+\sqrt{17}}{2}}</math> <math>(x^2-x-4)(x^2+x-3)</math>
    1 KB (196 words) - 21:49, 4 December 2016
  • ...ue of the difference between any two consecutive digits of <math>n</math> is at least <math>7</math> . Compute the number of <math>8</math>-digit yo-yos ...<math>7</math> if <math>n</math> is even. By symmetry, the desired answer is <math>2(a_8 + b_8 + c_8) - a_8</math>, to exclude the integers with leftmos
    2 KB (326 words) - 19:28, 10 March 2015
  • D(MP("A",v(0))--MP("B",v(1),N)--MP("C",v(2),N)--MP("D",v(3),SW)--MP("E",v(4))--MP("F",v(5))--cycle); D(v(0)--v(3));D(v(1)--v(5));D(v(1)--v(3));D(v(2)--v(4));
    3 KB (425 words) - 22:32, 5 December 2020
  • ...>BC</math>. <math>P</math> is on <math>MN</math> such that <math>N</math> is between <math>M</math> and <math>P</math>, and <math>m\angle MAN = m\angle pair A=(0,2), D=(0,0), C=(2,0), B=(2,2), M=(C+D)/2, N=(B+C)/2, P=8/3*(N-M)+M;
    2 KB (283 words) - 22:51, 13 April 2015
  • ...math>, which is also the area, so the answer is <math>\boxed{\frac {\sqrt {3}}{2}}</math>. ...tem reduces and we find that the desired sum is <math>\boxed{\frac {\sqrt {3}}{2}}</math>.
    2 KB (267 words) - 20:17, 4 December 2016
  • <math>ABCD</math> is a convex quadrilateral such that <math>|AB| = 5</math>, <math>|BC| = 17</ma ...(a proper divisor of <math>n</math> is a positive integer that divides but is not equal to <math>n</math>).
    4 KB (582 words) - 21:57, 8 May 2019
  • ...nd center <math>A</math> at points <math>P</math> and <math>D</math>. What is the distance from <math>P</math> to <math>\overline{AD}</math>? ...ac {16}{5} \qquad \textbf{(C)}\ \frac {13}{4} \qquad \textbf{(D)}\ 2\sqrt {3} \qquad \textbf{(E)}\ \frac {7}{2}</math>
    6 KB (1,026 words) - 22:35, 29 March 2023
  • <cmath>P(x) = x^5 + ax^4 + bx^3 + cx^2 + dx + e</cmath> has five distinct <math>x</math>-intercepts, one of which is at <math>(0,0)</math>. Which of the following [[coefficient]]s cannot be ze
    1 KB (210 words) - 21:12, 29 September 2021
  • ...a''' is a [[combinatorics |combinatorial]] result in [[group theory]] that is useful for counting the [[orbit]]s of a [[set]] on which a [[group]] [[grou ...e it is also called the '''Cauchy-Frobenius Lemma''', or '''the lemma that is not Burnside's'''. The lemma was (mistakenly) attributed to Burnside becau
    5 KB (757 words) - 18:11, 23 October 2023
  • ...m.''' In every finite group, the number of Sylow <math>p</math>-subgroups is equivalent to 1 (mod <math>p</math>). Evidently, a subset of <math>A \times T</math> is stable under this action if and only if <math>A= G</math>. Thus the fixed
    11 KB (2,071 words) - 12:25, 9 April 2019
  • ...alue of <math>x</math> has a prime factorization <math>a^cb^d.</math> What is <math>a + b + c + d?</math> ...\equiv 0 \pmod{5}</math>. Hence, we can substitute the quantity <math>11^{3} \cdot 7^{2}</math> into <math>y</math>. Doing so gets us:
    6 KB (914 words) - 11:07, 7 September 2023
  • River draws four cards from a standard 52 card deck of playing cards. Exactly 3 of them are 2’s. Find the probability River drew exactly one spade and on The probability is equal to the number of successful outcomes(<math>S</math>) divided by the n
    1 KB (223 words) - 19:43, 12 April 2012
  • Consider the triangular array of numbers with 0,1,2,3,... along the sides and interior numbers obtained by adding the two adjacen & & 3 & & 4 & & 4 & & 3 & & \\
    5 KB (682 words) - 09:45, 18 February 2022
  • The meaning of this page is to collect the problems posed there and save hints and solution suggestions ...do the wiki table formatting more elegantly, be my guest; after all, this is wikiiii.
    22 KB (3,358 words) - 15:17, 18 July 2017
  • Lemma 1: 2007 = 223 * 3^2 (trivial) For n = 3: n(3) = 6 * 20 - 12 * 4 + 8 = 80 = 2^3 * (3^2 + 1)
    2 KB (393 words) - 12:54, 11 August 2023
  • ...y, and <math>AB = 1</math>. The maximum possible length of <math>OB</math> is ...\ \sqrt{3} } \qquad \mathrm{(D) \ 2 } \qquad \mathrm{(E) \ \frac{4}{\sqrt{3}} } </math>
    1 KB (167 words) - 13:59, 5 July 2013
  • ...ld also try to strengthen in the areas you are not as good at. This guide is intended to help you get started. ...osttests to see if the books are on the right level for you. '''Alcumus''' is a good resource even if you do not own any of the AoPS books.
    13 KB (1,926 words) - 11:22, 30 November 2023
  • There are unique integers <math>a_{2},a_{3},a_{4},a_{5},a_{6},a_{7}</math> such that <cmath>\frac {5}{7} = \frac {a_{2}}{2!} + \frac {a_{3}}{3!} + \frac {a_{4}}{4!} + \frac {a_{5}}{5!} + \frac {a_{6}}{6!} + \frac {a_{7
    4 KB (595 words) - 01:43, 4 October 2023
  • ...Among all such triangles, the smallest possible value of <math>AC</math> is A=(5,12); B=origin; C=(10,0); D=(8.52071005917,3.55029585799);
    1 KB (173 words) - 14:35, 5 July 2013
  • ...probability that <math>P</math> lies inside one of the five small spheres is closest to ...qquad \mathrm{(B) \ }0.1 \qquad \mathrm{(C) \ }0.2 \qquad \mathrm{(D) \ }0.3 \qquad \mathrm{(E) \ }0.4 </math>
    3 KB (522 words) - 11:39, 3 October 2023
  • === Problem 3 === [[2008 IMO Problems/Problem 3 | Solution]]
    4 KB (613 words) - 00:30, 18 February 2021
  • ...math>4 \sin B + 3 \cos A = 1</math>. Then <math>\angle C</math> in degrees is Thus <math>\frac 12 = \sin A \cos B + \sin B \cos A</math>. This is the sine addition identity, so <math>\frac 12 = \sin (A + B) = \sin (180 -
    2 KB (296 words) - 10:21, 3 October 2023
  • ...thout rotating or reflecting, in position as below. Which of the rectangle is the top leftmost one? draw((0,5)--(0,7)--(3,7)--(3,5)--cycle);
    15 KB (2,222 words) - 10:40, 11 August 2020
  • ...in the [[prime factorization]] of <math>n!</math> and <math>S_p(n)</math> is the [[sum]] of the [[digit]]s of <math>n</math> when written in [[base]] <m ...ht\rfloor+\left\lfloor\frac{27}{2^2}\right\rfloor+\left\lfloor\frac {27}{2^3}\right\rfloor+\left\lfloor\frac{27}{2^4}\right\rfloor\\
    4 KB (699 words) - 17:55, 5 August 2023
  • ...angle <math>CAD</math> is twice angle <math>DAB</math>. If <math>AC/AD = 2/3</math>, then <math>CD/BD = m/n</math>, where <math>m</math> and <math>n</ma ...ath>. Then, it is given that <math>\cos 2\theta = \frac{AC}{AD} = \frac{2}{3}</math> and
    4 KB (662 words) - 00:51, 3 October 2023
  • A square with sides of length <math>1</math> is divided into two congruent trapezoids and a pentagon, which have equal area Then <math>2[I]+2[II] = [I]+3[II] \Longrightarrow [I]=[II]</math>. Let the shorter side of <math>I</math>
    2 KB (347 words) - 11:28, 2 December 2019
  • ...math> to mean <math>\frac {a+b}c</math>, where <math>c \neq 0</math>. What is the value of <center><math>\left[[60,30,90],[2,1,3],[10,5,15]\right]?</math></center>
    714 bytes (95 words) - 14:28, 5 July 2013
  • The addition below is incorrect. The display can be made correct by changing one digit <math>d</m + & 8 & 2 & 9 & 4 & 3 & 0 \\
    1 KB (223 words) - 13:59, 5 July 2013
  • ...h> be a sequence of positive integers. Let <math>m</math> be the number of 3-element subsequences <math>(a_i, a_j, a_k)</math> with <math>1 \le i < j < ...ue if and only if they belong to the same block. We can do this because it is unstrictly increasing. For example, if the sequence consists of 1000 ones,
    4 KB (703 words) - 12:45, 27 November 2017
  • ...h>MC = MF</math> if and only if <math>\angle GCD = \angle FDA</math>, that is, <math>\angle FDA + \angle CGF = 180^\circ</math>. Because quadrilateral <math>ABED</math> is cyclic, <math>\angle FDA = \angle ABE</math>. It follows that <math>MC = MF
    5 KB (820 words) - 02:39, 10 January 2023
  • Since all terms are homogeneous, we may assume WLOG that <math>a + b + c = 3</math>. ...- a)^2} = \sum \frac {a^2 + 6a + 9}{3a^2 - 6a + 9} = \sum \left(\frac {1}{3} + \frac {8a + 6}{3a^2 - 6a + 9}\right)</math>.
    6 KB (1,063 words) - 02:36, 9 August 2023
  • ...gular hexagon are written six nonnegative integers whose sum is 2003. Bert is allowed to make moves of the following form: he may pick a vertex and repla ...ginal numbers are <math>a,b,c,d,e,f</math>. Since <math>a+b+c+d+e+f</math> is odd, either <math>a+c+e</math> or <math>b+d+f</math> must be odd. WLOG let
    5 KB (739 words) - 13:39, 4 July 2013
  • ...th>5 \left( \dfrac{1}{AP} + \dfrac{1}{BQ} + \dfrac{1}{CR} \right) - \dfrac{3}{\min\{ AP, BQ, CR \}} = \dfrac{6}{r},</cmath> where <math>r</math> is the inradius and <math>P, Q, R</math> are the points of tangency of the inc
    4 KB (703 words) - 18:40, 3 January 2019
  • We claim that there is only one such sequence: <math>a_1=2, a_2=5, a_3=56, a_4=56\times 1400</math ...k that <math>(a_{k+1}-1)a_{k-1} \geq a_k^2(a_k - 1)</math> for <math>k=1,2,3</math>.
    3 KB (477 words) - 17:42, 8 August 2019
  • ...} + \frac {b}{\sqrt {b^2 + kca}} + \frac {c}{\sqrt {c^2 + kab}} \geq \frac{3}{\sqrt{1+k}}.</math></center> ...the function <math>f(x)=\frac{1}{\sqrt{x}}</math>. Note that this function is convex and monotonically decreasing which implies that if <math>a > b</math
    3 KB (453 words) - 00:12, 28 March 2021
  • ...that there are two or fewer people at the party whose departure leaves no 3-clique remaining. ...e exists only one 3-clique, remove anyone in that clique. (If there are no 3-cliques, we are done!) Otherwise, consider the following cases:
    3 KB (479 words) - 15:01, 23 November 2017
  • ...ee nonnegative integers <math>\{x,y,z\}</math> with <math>x < y < z</math> is called <i>historic</i> if <math>\{z - y,y - x\} = \{1776,2001\}</math>. Sh ..., y, or z position in its historic set (e.g. in <math>\{1,2,4\}</math>, 4 is in column z, 2 in column y, and 1 in column x).
    2 KB (494 words) - 17:34, 5 July 2020
  • ...PCE</math>, <math>PAF</math> are all equal. Prove that each of these areas is equal to the area of triangle <math>ABC</math> itself. ...(0,1,0)</math>, <math>C</math> is <math>(0,0,1)</math>, and <math>P</math> is <math>(p,q,r)</math>, with <math>p+q+r=1</math>.
    3 KB (565 words) - 23:25, 2 April 2012
  • Prove that there is no positive integer <math>n</math> such that, for <math>k = 1,2,\ldots,9</m Suppose that there is such a number <math>n</math>. Let <math>a</math> be the number of digits of
    3 KB (440 words) - 08:14, 1 April 2022
  • ...th>, we discard the second inequality and have that <math>3+2\sqrt2</math> is a lower bound for <math>r</math> ...nominators and keep the same value of <math>r</math>. Thus, <math>\boxed{m=3+2\sqrt2}</math>.
    2 KB (267 words) - 14:38, 1 September 2017
  • ...at neither <math>a^{p - 1} - 1</math> nor <math>(a + 1)^{p - 1} - 1</math> is divisible by <math>p^2</math>. ...math>(\mathbb{Z}/p^2\mathbb{Z})^{\times}</math> is cyclic, we know that it is isomorphic to <math>(\mathbb{Z}/(p^2-p)\mathbb{Z})^{+}</math>.
    3 KB (454 words) - 22:27, 16 October 2020
  • Is it possible to find 100 positive integers not exceeding 25,000, such that a The answer is yes.
    2 KB (443 words) - 13:08, 17 August 2011
  • ...nctions <math>f: (0, \infty) \mapsto (0, \infty)</math> (so <math>f</math> is a function from the positive real numbers) such that ...he only possible solution is <math>f(1)=1</math> since <math>f(1)=0</math> is impossible.
    4 KB (661 words) - 01:10, 19 November 2023
  • ..."setting the agenda for the twentieth century". (Devlin 2003, pp. 2&ndash;3) These problems he believed to be the most significant and important unsolv ...y hard to isolate one problem that captures the program." (Devlin 2003, p. 3)
    13 KB (1,969 words) - 17:57, 22 February 2024
  • ...he circle on diameter <math>AB</math>. Prove that the line <math>AB</math> is tangent to the circle on diameter <math>CD</math> if and only if the lines ...hat the length of the perpendicular from <math>M</math> to <math>CD</math> is <math>\frac{1}{2}AB</math>. Let the foot of the perpendicular from <math>C<
    4 KB (750 words) - 23:49, 29 January 2021
  • ...to the sides (i.e., signed lengths of the pedal lines from <math>O</math>) is: c=(1,3);
    4 KB (723 words) - 01:45, 18 February 2021
  • is a union of disjoint intervals, the sum of whose length is <math>1988</math>. ...ve that the solution set to the inequality <math>f(x)\ge\frac{5}{4}</math> is the union of the intervals <math>(n,r_n]</math> (since if <math>f(x)<\frac{
    3 KB (518 words) - 11:36, 30 January 2021
  • ...>, the intersection of circle <math>A</math> with the line <math>AB</math> is the same as the intersection of circle <math>B</math> with the line <math>A ...<math>P</math> to <math>CD</math>, we have that the circle <math>P</math> is tangent to <math>CD</math>. From the conditions <math>AP=h+AD</math> and <
    4 KB (771 words) - 11:57, 30 January 2021
  • A '''Farey sequence''' of order <math>n</math> is the sequence of all completely reduced fractions between 0 and 1 where, whe <math>F_3=\{0/1, 1/3, 1/2, 2/3, 1/1\}</math>
    1 KB (199 words) - 04:16, 30 January 2021
  • What is the value of the sum <math>S=\sum_{k=0}^{49}(-1)^k\binom{99}{2k}=\binom{99} ...h> <math>\binom{99}{0}i^0+\binom{99}{1}i^1+\binom{99}{2}i^2+\binom{99}{3}i^3+\binom{99}{4}i^4+\cdots +\binom{99}{98}i^{98}</math>.
    1 KB (173 words) - 19:52, 30 December 2020
  • ...</math> has the property that none of its members is 3 times another. What is the largest number of members such a subset can have? ...s long as no integer between <math>11</math> and <math>33</math> inclusive is within the set. This provides a total of <math>100 - 34 + 1</math> = 67 sol
    2 KB (285 words) - 19:25, 25 September 2020
  • For <math>i\in\{1,2,3,\ldots,10\},</math> suppose Person <math>i</math> picks the number <math>a_ ...sets of equations are independent. The set that involves <math>a_6</math> is
    3 KB (402 words) - 23:17, 23 September 2023
  • ...ath>BD</math> is line <math>DN</math>. Since the pairwise radical axes of 3 circles are concurrent, we have <math>AM,DN,XY</math> are concurrent as des .... Thus, <math>N'</math> and <math>N''</math> are distinct, as none of them is <math>N</math>. Hence, by Power of a Point, <cmath>ZM * ZA = ZP * ZT = ZN''
    5 KB (847 words) - 19:03, 12 October 2021
  • ...mps are off. We consider sequences of steps: at each step one of the lamps is switched (from on to off or from off to on). ...ff, but where none of the lamps <math>n + 1</math> through <math>2n</math> is ever switched on.
    4 KB (677 words) - 01:10, 19 November 2023
  • '''(ii)''' Prove that equality is achieved for infinitely many triples of rational numbers <math>x</math>, <m ...ath>\mathbb{Q}/\{1\}</math> to <math>\mathbb{Q}/\{-1\}</math>, the problem is equivalent to showing that
    3 KB (593 words) - 01:09, 19 November 2023
  • ...th>5 \left( \dfrac{1}{AP} + \dfrac{1}{BQ} + \dfrac{1}{CR} \right) - \dfrac{3}{\min\{ AP, BQ, CR \}} = \dfrac{6}{r},</cmath> where <math>r</math> is the inradius and <math>P, Q, R</math> are the points of tangency of the inc
    3 KB (495 words) - 19:02, 18 April 2014
  • We claim that <math>n = 1999</math> is the smallest such number. For <math>n \le 1998</math>, we can simply color ...i == 0 || j == 9) && !(j-i == 9)) fill(shift(i,j)*unitsquare,rgb(0.3,0.3,0.3));
    2 KB (382 words) - 13:37, 4 July 2013
  • ...s through <math>A_k</math> and <math>A_{k + 1},</math> where <math>A_{n + 3} = A_{n}</math> for all <math>n \ge 1</math>. Prove that <math>\omega_7 = \ <math>O</math> is the intersection of the perpendicular bisectors of <math>\overline{A_1A_2},
    3 KB (609 words) - 09:52, 20 July 2016
  • ...hat if <math>P_{1986}=P_0</math>, then the triangle <math>A_1A_2A_3</math> is equilateral. ...math>A_3</math> the rotation sends <math>P</math> to <math>e^{\frac{4i\pi}{3}}(P-a)+a</math>. Thus the result of all three rotations sends <math>P</mat
    2 KB (345 words) - 00:04, 30 January 2021
  • ...ne, with justification, the smallest integer <math>n</math> for which this is possible. We claim that <math>n=23</math> is the minimum. Consider the following construction (replacing colors with num
    5 KB (841 words) - 17:19, 5 May 2022
  • ...ates. Michael is their oldest child, and Wendy their oldest daughter. Tony is the youngest child. Twins Joshua and Alexis are <math>12</math> years old. (Michael is still young enough to get the children's rate), and family memberships
    71 KB (11,749 words) - 01:31, 2 November 2023
  • Find the largest [[prime]] number less than <math>2008</math> that is a divisor of some integer in the infinite ...r, \left\lfloor \frac{2008^2}{2} \right\rfloor, \left\lfloor \frac{2008^3}{3}\right\rfloor, \left\lfloor \frac{2008^4}{4} \right\rfloor, \cdots</cmath>
    4 KB (571 words) - 21:21, 22 November 2018
  • <center><math>{1, 2, 3, . . . , 4n}</math></center> element which is the [[arithmetic mean]] of all the elements in that subset?
    1 KB (232 words) - 21:20, 22 November 2018
  • ...difference of the areas of <math>P_{10}</math> and <math>P_{\infty}</math> is written as a fraction <math>\frac{x}{y}</math> in lowest terms, calculate t ...our [[trapezoid]] be <math>P_1 = ABCD</math> (and <math>[ABCD] = \frac{12(3+13)}{2} = 96</math>); then [[without loss of generality]] construct diagona
    4 KB (685 words) - 14:39, 7 October 2017
  • ...the polynomial remainder when <math>p(x)</math> is divided by <math>x^4+x^3+2x^2+x+1</math>. Find the remainder when <math>|r(2008)|</math> is divided by <math>1000</math>.
    3 KB (560 words) - 19:49, 23 November 2018
  • ...that Skipper can reach? Calculate an approximate answer by using <math>\pi=3.14</math> or <math>\pi=22/7</math>. ...6}</math>. Adding, we get <math>\frac{8}{3}\pi+\frac{\pi}{6}+\frac{\pi}{6}=3\pi</math>. Then approximate.
    854 bytes (145 words) - 23:28, 21 October 2022
  • ...systems as well as other Unix systems. The third was tested on Mac OS 11.3 "Big Sur"''' .../project/showfiles.php?group_id=120000 here] (at the time of this edit, it is version 2.86) by selecting the latest version and downloading the asymptote
    5 KB (732 words) - 00:47, 13 December 2023
  • <math>ABC</math> is a triangle, the bisector of angle <math>A</math> meets the circumcircle of ...ath> and similarly for the other triangles. Thus, the area of the hexagon is equal to <math>[ABC]+\sum\frac{1}{4}a^2\left(\frac{2r}{b+c-a}\right)</math>
    3 KB (568 words) - 11:50, 30 January 2021
  • ...>2007</math>, and the arithmetic mean of <math>m</math> and <math>n</math> is less than <math>2007</math>. How many pairs <math>(m, n)</math> satisfy the ...>\mathrm{(A)}\,0\quad\mathrm{(B)}\,1\quad\mathrm{(C)}\,2\quad\mathrm{(D)}\,3\quad\mathrm{(E)}\,4\quad\mathrm{(F)}\,5\quad\mathrm{(G)}\,6\quad\mathrm{(H)
    966 bytes (152 words) - 20:24, 10 January 2019
  • ...re edges, the greatest common divisor of the integers labeling those edges is equal to 1. ...s from the set <math>V</math> if their exists an edge sorrounding it which is labelled. Additionally, we shall use up the numbers <math>1,2,...,k</math>
    4 KB (668 words) - 17:45, 30 January 2021
  • ...2</math> or <math>p \equiv 1 \pmod{4}</math>; and that this representation is unique. The theorem is a starting point in the investigation of [[binary quadratic forms]]. Histo
    4 KB (612 words) - 12:10, 30 May 2019
  • ...e circle; if it is on an odd-numbered point, it moves one point, and if it is on an even-numbered point, it moves two points. If the bug begins on point <math> \mathrm{(A) \ 1 } \qquad \mathrm{(B) \ 2 } \qquad \mathrm{(C) \ 3 } \qquad \mathrm{(D) \ 4 } \qquad \mathrm{(E) \ 5 } </math>
    1 KB (183 words) - 21:54, 2 August 2016
  • ...ath> at <math>A</math>. The number of degrees in minor arc <math>AD</math> is label("$D$", origin+1*dir(36+72*3), dir(origin--origin+1*dir(36+72*3)));
    1 KB (201 words) - 16:29, 12 March 2024
  • ...bisects that diagonal. The number of unit cubes that the plane intersects is ...ac 32,\frac 32\right)</math>. The plane that passes through this point and is orthogonal to the diagonal has the equation <math>x+y+z=\frac 92</math>.
    5 KB (828 words) - 05:52, 30 September 2023
  • ...<math>\triangle ABC</math> is parallel to <math>\overline{DE}</math>. What is the area of <math>\triangle BDE</math>? <math>\mathrm {(A)} \sqrt{2}\qquad \mathrm {(B)} \sqrt{3}\qquad \mathrm {(C)} 2\qquad \mathrm {(D)} \sqrt{5}\qquad \mathrm {(E)} \sq
    3 KB (470 words) - 19:46, 17 July 2023
  • <center><math>x+y+z=3</math>, <math>x^2+y^2+z^2=3</math>,
    5 KB (888 words) - 08:18, 22 April 2024
  • ...c_1</math>. Thus <math>a_1^2+b_1^2+c_1^2=4a_1^2b_1^2</math>. Since the LHS is divisible by four, all the variables are divisible by 4, and we must do thi ...}</math>. But for this to be true, <math>c^2\equiv 3\bmod{4}</math>, which is an impossibility. Thus there are no non-zero solutions when <math>a^2\equiv
    1 KB (214 words) - 00:00, 10 October 2020
  • fillsq(0,3);fillsq(1,3);fillsq(4,3);fillsq(5,3); draw((3,1)--(3,3)--(7,3)--(7,1)--cycle,black+1);
    4 KB (660 words) - 19:21, 23 November 2016
  • .../math>, <math>X_1=1</math>, <math>X_{n+1}=X_n+2X_{n-1}</math> <math>(n=1,2,3,\dots),</math></center> ...math>, <math>Y_1=7</math>, <math>Y_{n+1}=2Y_n+3Y_{n-1}</math> <math>(n=1,2,3,\dots)</math>.</center>
    2 KB (342 words) - 18:54, 3 July 2013
  • ...C 12A Problems|2002 AMC 12A #3]] and [[2002 AMC 10A Problems|2002 AMC 10A #3]]}} If the order in which the exponentiations are performed is changed, how many other values are possible?
    2 KB (224 words) - 12:28, 8 November 2021
  • therefore the answer is <math>\boxed{\textbf{(E) } \text{infinitely many}}</math>. ...many numbers <math>m</math> that can satisfy the inequality. So the answer is <math>\boxed{\textbf{(E) } \text{infinitely many}}</math>.
    2 KB (258 words) - 04:58, 21 July 2022
  • ...ons <math>a</math>, <math>b</math> the difference <math>f(a) - f(b)</math> is a multiple of <math>m!</math>. ...Using the fact that <math>f(v_t)\equiv t\pmod{m!}</math>, we find the sum is congruent to <math>0+1+\cdots+m!-1=\frac{m!(m!-1)}{2}</math>.
    2 KB (416 words) - 20:01, 19 May 2023
  • ...h>b</math>, <math>c</math>, and <math>d</math> are positive integers, what is the smallest possible value of <math>a+b+c+d</math>? real a = 3, b = 1, c = 9, d = 3;
    7 KB (1,143 words) - 21:25, 20 December 2020
  • ...th> and <math>BY</math> bisects angle <math>B</math>. Angle <math>A</math> is <math>60^{\circ}</math>. <math>AB + BX = AY + YB</math>. Find all possible dotfactor = 3;
    4 KB (692 words) - 15:01, 15 May 2024
  • ...</math> and multiplies them. What is the probability that Tamika's result is greater than Carlos' result? ...\dfrac{1}{2} \qquad \text{(D)}\ \dfrac{1}{3} \qquad \text{(E)}\ \dfrac{2}{3}</math>
    2 KB (242 words) - 19:53, 31 October 2016
  • have divided the number by 2 to get the correct answer. What is the correct ==Problem 3==
    13 KB (1,821 words) - 22:18, 5 December 2023
  • ...ane having the common center. The order in which the composition is taken is not important. The transformation is linear and transforms any given object into an object homothetic to given.
    28 KB (4,863 words) - 00:29, 16 December 2023
  • ...he line <math>y=x</math>. If <math>(2,3)</math> is in <math>S</math>, what is the smallest number of points in <math>S</math>? ...th>8</math> points satisfy all of the symmetry conditions. Thus the answer is <math>\boxed{\mathrm{(D)}\ 8}</math>.
    876 bytes (140 words) - 20:35, 30 December 2014
  • ...on they come at different times to claim their prizes, and each assumes he is the first to arrive. If each takes what he believes to be the correct share ...cation, it does not matter what order the three come in.) Hence the answer is <math>\boxed{\mathrm{(D)}\ \dfrac{5}{18}}</math>.
    1 KB (213 words) - 10:16, 4 July 2013
  • ...ong the simplest examples of a [[fractal]]. [[Topology|Topologically]], it is a [[closed set]], and also a [[perfect set]]. Despite containing an [[uncou ...th>, dividing the [[interval]] into two intervals of length <math>\frac{1}{3}</math>. Then remove the middle third of the two remaining segments, and r
    3 KB (413 words) - 16:32, 18 June 2020
  • (''Hojoo Lee'') Let <math>n \geq 3</math> be an integer. Let <math>t_1, t_2, \dots , t_n</math> be positive re For <math>n=3</math>, suppose (for sake of contradiction) that <math>t_3 = t_2 + t_1 + k<
    2 KB (322 words) - 00:54, 19 November 2023
  • Which of the following statements is false? ...are equal. Thus equilateral triangles are equiangular, and <math>C</math> is true.
    2 KB (259 words) - 14:28, 13 February 2019
  • <math>2\sum_{k=1}^{n}a_{k}^{2}\ge 2(a_{1}a_{2}+a_{2}a_{3}+\cdots+a_{n-1}a_{n}+a_{n}a_{1})</math> Which is always true.
    464 bytes (82 words) - 15:18, 23 May 2009

View (previous 500 | next 500) (20 | 50 | 100 | 250 | 500)